You are on page 1of 97

2009 FRM Practice Exams 

2009 FRM Practice Exams 
 
 
Table of Contents 
2009 FRM Level I Practice Exam ‐ Candidate Answer Sheet ............................................. 3 
2009 FRM Level I Practice Exam – Questions  ................................................................... 5 
2009 FRM Level I Practice Exam – Answer Key ............................................................... 19 
2009 FRM Level I Practice Exam – Answers & Explanations ........................................... 21 
2009 FRM Full Exam FRM Practice Exam I – Candidate Answer Sheet  .......................... 47 
2009 FRM Full Exam FRM Practice Exam I – Questions .................................................. 49 
2009 FRM Full Exam FRM Practice Exam I – Answer Key  ............................................... 65 
2009 FRM Full Exam FRM Practice Exam I – Answers & Explanations  ........................... 67 
2009 FRM Full Exam FRM Practice Exam II – Candidate Answer Sheet  ......................... 99 
2009 FRM Full Exam FRM Practice Exam II – Questions ............................................... 101 
2009 FRM Full Exam FRM Practice Exam II – Answer Key  ............................................ 117 
2009 FRM Full Exam FRM Practice Exam II – Answers & Explanations  ........................ 119 
 
 
Introduction 
The FRM exam is a practice‐oriented examination. Its questions are derived from a combination 
of theory, as set forth in the core readings, and “real‐world” work experience. Candidates are 
expected to understand risk management concepts and approaches and how they would apply 
to a risk manager’s day‐to‐day activities.   
The  FRM  examination  is  also  a  comprehensive  examination,  testing  a  risk  professional  on  a 
number of risk management concepts and approaches. It is very rare that a risk manager will be 
faced with an issue that can immediately be slotted into one category. In the real world, a risk 
manager  must  be  able  to  identify  any  number  of  risk‐related  issues  and  be  able  to  deal  with 
them effectively. 
The 2009 FRM Practice Exams have been developed to aid candidates in their preparation for 
the  FRM  Examination  in  November  2009.    These  practice  exams  are  based  on  a  sample  of 
questions from the 2007 FRM Examination and are representative of the questions that will be 
in  the  2009  FRM  Examination.    Wherever  necessary  and  possible,  questions,  answers  and 
references have been updated to better reflect the topics and core readings listed in the 2009 
FRM Examination Study Guide. 
The 2009 FRM Level I Practice Exam and the FRM Full Exam Practice Exams I and II contain 40 
and  50  multiple‐choice  questions,  respectively.    Note  that  the  2009  FRM  Level  I  and  Full 
Examination  will  consist  of  a  morning  and  afternoon  session  containing  50  and  70  multiple‐
choice  questions,  respectively.    The  practice  exams  were  designed  to  be  shorter  to  allow 
candidates to calibrate their preparedness without being overwhelming. 

 
Copyright © 2009 Global Association of Risk Professionals         1    
All rights reserved.
2009 FRM Practice Exams 

The 2009 FRM Practice Exams do not necessarily cover all topics to be tested in the 2009 FRM 
Examination.    For  a  complete  list  of  topics  and  core  readings,  candidates  should  refer  to  the 
2009  FRM  Examination  Study  Guide.    Core  readings  were  selected  by  the  FRM  Committee  to 
assist candidates in their review of the subjects covered by the exam.  Questions for the FRM 
examination  are  derived  from  the  “core”  readings.  It  is  strongly  suggested  that  candidates 
review these readings in depth prior to sitting for the exam. 

Suggested Use of Practice Exams 
To maximize the effectiveness of the practice exams, candidates are encouraged to follow 
these recommendations: 
• Plan a date and time to take each practice exam.  Set dates appropriately to give 
sufficient study/review time between each practice exam and prior to the actual exam. 
• Simulate the test environment as closely as possible. 
o Take each practice exam in a quiet place. 
o Have only the practice exam, candidate answer sheet, calculator, and writing 
instruments (pencils, erasers) available. 
o Minimize possible distractions from other people, cell phones and study 
material. 
o Allocate 90 minutes for each practice exam and set an alarm to alert you when 
90 minutes have passed.  Complete each exam but note the questions answered 
after the 90 minute mark. 
o Follow the FRM calculator policy.  You may only use a Texas Instruments BA II 
Plus (including the BA II Plus Professional) calculator or a Hewlett Packard 12C 
(including the HP 12C Platinum) calculator. 
• After completing each practice exam, 
o Calculate your score by comparing your answer sheet with the practice exam 
answer key.  Only include questions completed in the first 90 minutes. 
o Use the practice exam Answers & Explanations to better understand correct and 
incorrect answers and to identify topics that require additional review.  Consult 
referenced core readings to prepare for exam. 
o Pass/fail status for the actual exam is based on the distribution of scores from all 
candidates, so use your scores only to gauge your own progress and 
preparedness. 

 
Copyright © 2009 Global Association of Risk Professionals         2    
All rights reserved.
2009 FRM Practice Exams 

2009 FRM Level I Practice Exam  
Candidate Answer Sheet 
 
1. a. b. c. d. 21. a. b. c. d.
2. a. b. c. d. 22. a. b. c. d.
3. a. b. c. d. 23. a. b. c. d.
4. a. b. c. d. 24. a. b. c. d.
5. a. b. c. d. 25. a. b. c. d.
6. a. b. c. d. 26. a. b. c. d.
7. a. b. c. d. 27. a. b. c. d.
8. a. b. c. d. 28. a. b. c. d.
9. a. b. c. d. 29. a. b. c. d.
10. a. b. c. d. 30. a. b. c. d.
11. a. b. c. d. 31. a. b. c. d.
12. a. b. c. d. 32. a. b. c. d.
13. a. b. c. d. 33. a. b. c. d.
14. a. b. c. d. 34. a. b. c. d.
15. a. b. c. d. 35. a. b. c. d.
16. a. b. c. d. 36. a. b. c. d.
17. a. b. c. d. 37. a. b. c. d.
18. a. b. c. d. 38. a. b. c. d.
19. a. b. c. d. 39. a. b. c. d.
20. a. b. c. d. 40. a. b. c. d.

 
Copyright © 2009 Global Association of Risk Professionals         3    
All rights reserved.
2009 FRM Practice Exams 

This page is intentionally left blank.  
 

 
Copyright © 2009 Global Association of Risk Professionals         4    
All rights reserved.
2009 FRM Practice Exams 

2009 FRM Level I Practice Exam  
Questions 
 
 
 
1. To hedge against future, unanticipated, and significant increases in borrowing rates, which of the 
following alternatives offers the greatest flexibility for the borrower? 
 
a. Fixed for floating swap 
b. Interest rate collar 
c. Interest rate floor 
d. Call swaption 
 
 
 
 
 
2. An investment bank uses the Exponentially Weighted Moving Average (EWMA) technique with 
lambda of 0.9 to model the daily volatility of a security. The current estimate of the daily volatility is 
1.5%. The closing price of the security is USD 20 yesterday and USD 18 today. Using continuously‐
compounded returns, what is the updated estimate of the volatility? 

a. 5.44% 
b. 3.62% 
c. 2.96% 
d. 1.31% 
 
 
 
 
 
3. Consider two stocks A and B.  Assume their annual returns are jointly normally distributed, the 
marginal distribution of each stock has mean 2% and standard deviation 10%, and the correlation is 
0.9.  What is the expected annual return of stock A if the annual return of stock B is 3%? 
 
a. 2.9% 
b. 2% 
c. 1.1% 
d. 4.7% 

 
Copyright © 2009 Global Association of Risk Professionals         5    
All rights reserved.
2009 FRM Practice Exams 

4. In pricing a derivative using the Monte Carlo method, we need to simulate a reasonable number of 
paths for the price of the underlying asset.  Suppose we use a simple model for the return of the 
underlying asset: 
 
y(t) = drift*Δt + vol * √ Δt * e(t), and e(t) is distributed ~ N(0,1), 
 
where drift and vol are known parameters and Δt is the step size. 
 
The generation of each path requires a number of steps. Which of the following describes the 
correct procedure? 
 
a. Generate a random number from a normal distribution N(0,1), use the inverse normal function 
to get e(t), which will be fed into the model to get y(t). Repeat the same procedure until you get 
the full desired path. 
b. Generate a random number from a normal distribution N(0,1), use the cumulative normal 
function to get e(t), which will be fed into the model to get y(t). Repeat the same procedure 
until you get the full desired path. 
c. Generate a random number from a uniform distribution defined in [0,1], use the inverse 
cumulative normal function to get e(t), which will be fed into the model to get y(t). Repeat the 
same procedure until you get the full desired path. 
d. Generate a random number from a uniform distribution defined in [0,1], use the cumulative 
normal function to get e(t), which will be fed into the model to get y(t). Repeat the same 
procedure until you get the full desired path. 
 
 
 
 
 
 
5. A risk manager estimates the daily variance (ht) using a GARCH model on daily returns (rt):

ht = α0 + α1r2t-1 + βht-1

Assume the model parameter values are α0 = 0.005, α = 0.04, β = 0.94.  The long‐run annualized 


volatility is approximately: 

a. 25.00% 
b. 13.54% 
c. 72.72% 
d. 7.94% 
 

 
Copyright © 2009 Global Association of Risk Professionals         6    
All rights reserved.
2009 FRM Practice Exams 

6. A single stock has a price of USD 10 and a current daily volatility of 2%. Using the delta‐normal 
method, the VaR at the 95% confidence level of a long at‐the‐money call on this stock over a 1‐day 
holding period is approximately: 
 
a. USD 0.23 
b. USD 1.645 
c. USD 0.33 
d. USD 0.16 
 
 
 
7. A portfolio consists of two zero coupon bonds, each with a current value of USD 10. The first bond 
has a modified duration of 1 year and the second has a modified duration of 9 years. The yield curve 
is flat and all yields are 5%. Assume all moves of the yield curve are parallel shifts. Given that the 
daily volatility of the yield is 1%, which of the following is the best estimate of the portfolio daily VaR 
at the 95% confidence level? 
 
a. USD 2.33 
b. USD 1.65 
c. USD 0.82 
d. USD 1.16 
 
 
 
8. Consider the following three methods of estimating the P&L of a bullet bond: full repricing, duration 
(PV01), and duration plus convexity.  Ranking the estimated P&L impact of a large negative yield 
shock from the lowest P&L impact to the highest P&L impact, what is the ranking of the methods to 
estimate the P&L impact?  
 
a. Duration plus convexity, duration, full repricing 
b. Full repricing, duration plus convexity, duration 
c. Duration, duration plus convexity, full repricing 
d. Duration, full repricing, duration plus convexity  
   
 
 
9. Consider a position in a 5‐year receive‐fixed swap that makes annual payments on a USD 100 million 
notional.  The floating leg has just been reset.  The term structure is flat at 5%, the Macaulay 
duration of a 5‐year par bond is 4.5 years, and the annual volatility of yield changes is 100bp.  Your 
best estimate of the swap’s VaR with 95% confidence over the next month is: 
 
a. USD 1.6 million 
b. USD 2.0 million 
c. USD 5.5 million 
d. USD 7.1 million 

 
Copyright © 2009 Global Association of Risk Professionals         7    
All rights reserved.
2009 FRM Practice Exams 

10. If the gold lease rate is higher than the risk‐free rate, what is the market structure of the forward 
market for gold? 
  
a. Contango 
b. Backwardation 
c. Inversion 
d. Need more information to determine 
 
 
 
 
11. The price of a 3‐year zero coupon government bond is 85.16.  The price of a similar 4‐year bond is 
79.81.  What is the one‐year implied forward rate from year 3 to year 4? 
 
a. 5.4% 
b. 5.5% 
c. 5.8% 
d. 6.7% 
 
 
 
 
12. A portfolio manager has a bond position worth USD 100 million.  The position has a modified 
duration of 8 years and a convexity of 150 years.  Assume that the term structure is flat.  By how 
much does the value of the position change if interest rates increase by 25 basis points? 
 
a. USD ‐1,953,125 
b. USD ‐1,906,250 
c. USD ‐2,046,875 
d. UDS ‐2,187,500 
 
 
 
 
13. A firm is going to buy 10,000 barrels of West Texas Crude Oil.  It plans to hedge the purchase using 
the Brent Crude futures contract.  The correlation between the spot and futures prices is 0.72.  The 
volatility of the spot price is 0.35 per year.  The volatility of the Brent Crude futures price is 0.27 per 
year.  What is the hedge ratio for the firm? 
 
a. 0.5554 
b. 0.9333 
c. 1.2099 
d. 0.8198 

 
Copyright © 2009 Global Association of Risk Professionals         8    
All rights reserved.
2009 FRM Practice Exams 

14. It is June 2nd and a fund manager with USD 10 million invested in government bonds is concerned 
that interest rates will be highly volatile over the next three months.  The manager decides to use 
the September Treasury bond futures contract to hedge the value of the portfolio.  The current 
futures price is 95.0625.  Each contract is for the delivery of USD 100,000 face value of bonds.  The 
duration of the manager’s bond portfolio in three months will be 7.8 years.  The cheapest to deliver 
bond in the Treasury bond futures contract is expected to have a duration of 8.4 years at maturity of 
the contract.  At the maturity of the Treasury bond futures contract, the duration of the underlying 
benchmark Treasury bond is 9 years. What position should the fund manager undertake to mitigate 
his interest rate risk exposure? 
 
a. Short 94 contracts 
b. Short 98 contracts 
c. Short 105 contracts 
d. Short 113 contracts 
 
 
 
 
 
15. A bond trader has bought a position in Treasury Bonds with a 4% annual coupon rate on February 
15, 2015. The DV01 of the position is USD 80,000. The trader decides to hedge his interest rate risk 
with the 4.5% coupon rate Treasury Bonds maturing on May 15, 2017 which has a DV01 of .076 per 
USD 100 face value. To implement this hedge, approximately what face amount of the 4.5% 
Treasury bonds maturing on May 15, 2017 should the trader sell? 
 
a. USD 80,000 
b. USD 10,500,000 
c. USD 80,000,000 
d. USD 105,000,000 
 
 
 
 
 
16. Suppose that A and B are random variables, each follows a standard normal distribution, and the 
covariance between A and B is 0.35. What is the variance of (3A + 2B)? 
 
a. 15.10 
b. 14.47 
c. 9.20 
d. 17.20 

 
Copyright © 2009 Global Association of Risk Professionals         9    
All rights reserved.
2009 FRM Practice Exams 

17. Consider a stock price S that follows a geometric Brownian motion dS = μ S dt + β S dz, with β


strictly positive and μ a fixed value.  Which of the following statements is true? 
 
a. If the drift μ is negative, the price one year from now will be below today’s price. 
b. The instantaneous rate of return on the stock follows a uniform distribution. 
c. The stock price S follows a lognormal distribution. 
d. This model imposes mean reversion. 
 
 
 
 
 
18. The joint probability distribution of random variables X and Y is given by f(x,y) = kxy for x = 1, 2, 3, y = 
1, 2, 3, and k is a positive constant. What is the probability that X + Y will exceed 5? 
 
a. 1/9  
b. 1/4 
c. 1/36  
d. Cannot be determined 
 
 
 
 
 
19. Which of the following statements regarding Hypothesis Testing is incorrect? 
 
a. Hypothesis testing is used to make inferences about the parameters of a given population on 
the basis of statistics computed for a sample that is drawn from that population. 
b. Type II error refers to the failure to reject the null hypothesis when it is actually false. 
c. The p‐value decision rule is to reject the null hypothesis if the p‐value is greater than the 
significance level. 
d. All else being equal, the decrease in the chance of making a Type I error comes at the cost of 
increasing the probability of making a Type II error. 
 
 
 
 
 
20. If stock returns are independently identically normally distributed and the annual volatility is 30%, 
then the daily VaR at the 99% confidence level of a stock market portfolio is approximately: 
 
a. 2.41% 
b. 3.11% 
c. 4.40% 
d. 1.89% 

 
Copyright © 2009 Global Association of Risk Professionals         10    
All rights reserved.
2009 FRM Practice Exams 

21. The current value of the S&P 500 index is 1457, and each S&P futures contract is for delivery of USD 
250 times the index.  A long‐only equity portfolio with market value of USD 300,100,000 has beta of 
1.1.  To reduce the portfolio beta to 0.75, how many S&P futures contract should you sell? 

 
a. 618 contracts 
b. 288 contracts 
c. 574 contracts 
d. 906 contracts 
 
 
 
 
The following information should be used for the next two questions. 
On January 1, a risk manager observes that the 1‐year continuously compounded interest rate is 5% 
and storage costs of a commodity product A is USD 0.05 per quarter (payable at each quarter end).  
He further observes the following forward prices for product A: 
 
March      5.35 
June      5.90 
September    5.30 
December     5.22 
 
 
22. Given the following explanation of supply and demand for commodity product A how would you 
best describe its forward price curve from June to December? 
 
  Market description      Explanation 
a. Backwardation        Excess demand for A in early summer 
b. Backwardation        Supply is expected to decline after summer 
c. Contango        Excess demand for A in early summer 
d. Contango        Supply is expected to decline after summer 
 
 
 
 
23. What is the annualized rate of return earned on a cash‐and‐carry trade entered into in March and 
closed out in June? 
 
a. 8.9% 
b. 9.8 % 
c. 35.7% 
d. 39.1% 

 
Copyright © 2009 Global Association of Risk Professionals         11    
All rights reserved.
2009 FRM Practice Exams 

24. An investor sells a June 2008 call of ABC Limited with a strike price of USD 45 for USD 3 and buys a 
June 2008 call of ABC Limited with a strike price of USD 40 for USD 5. What is the name of this 
strategy and the maximum profit and loss the investor could incur? 
 
a. Bear Spread, Maximum Loss USD 2, Maximum Profit USD 3 
b. Bull Spread, Maximum Loss Unlimited, Maximum Profit USD 3 
c. Bear Spread, Maximum Loss USD 2, Maximum Profit Unlimited 
d. Bull Spread, Maximum Loss USD 2, Maximum Profit USD 3 
 
 
 
 
 
 
25. Which of the following problems are NOT inherent disadvantages of the historical simulation 
approach to estimating VaR? 
 
I. It gives too little weight to more recent observations 
II. For long‐only portfolios, it is likely to understate VaR following a recent structural increase 
in volatilities 
III. It always ignores the fat tails present in the distribution of returns on many financial assets 
IV. Because of the delta approximation, it inadequately measures the risk of nonlinear 
instruments 
 
a. I and II only 
b. II only 
c. I, III and IV only 
d. III and IV only 
 
 
 
 
 
 
26. A bank holds USD 60 million worth of 10‐year 6.5% coupon bonds that are trading at a clean price of 
101.82. The bank is worried by the exposure due to these bonds but cannot unwind the position for 
fear of upsetting the client. Therefore, it purchases a total return swap (TRS) in which it receives 
annual Libor + 100 bps in return for the mark‐to‐market return on the bond.  For the first year, the 
Libor sets at 6.25% and by the end of the year the clean price of the bonds is at 99.35. The net 
receipt/payment for the bank in the total return swap will be: 
 
a. Receive USD 2.23 million  
b. Receive USD 1.93 million 
c. Pay USD 1.93 million 
d. Pay USD 2.23 million 
 

 
Copyright © 2009 Global Association of Risk Professionals         12    
All rights reserved.
2009 FRM Practice Exams 

27. Which of the following trade(s) contain basis risk? 
 
I. Long 1,000 lots Nov 07 ICE Brent Oil contracts and short 1,000 lots Nov 07 NYMEX WTI 
Crude Oil contracts 
II. Long 1,000 lots Nov 07 ICE Brent Oil contracts and long 2,000 lots Nov 07 ICE Brent Oil at‐
the‐money put 
III. Long 1,000 lots Nov 07 Brent Oil contracts and short 1,000 lots Dec 07 ICE Brent Oil 
contracts 
IV. Long 1,000 lots Nov 07 ICE Brent Oil contracts and short 1,000 lots Dec 07 NYMEX WTI 
Crude Oil contracts 
 
a. I & III 
b. II &IV 
c. III & IV 
d. I, III & IV 
 
 
 
 
 
 
28. According to put‐call parity, buying a put option on a stock is equivalent to: 
 
a. Buying a call option and buying the stock with funds borrowed at the risk‐free rate. 
b. Selling a call option and buying the stock with funds borrowed at the risk‐free rate. 
c. Buying a call option, selling the stock and investing the proceeds  at the risk‐free rate. 
d. Selling a call option, selling the stock and investing the proceeds at the risk‐free rate. 
 
 
 
 
 
 
 
29. A 3 month futures contract on an equity index is currently priced at USD 1000, the underlying index 
stocks are valued at USD 990 and pay dividends at a continuously‐compounded rate of 2 percent 
and the current continuously compounded risk‐free rate is 4 percent. The potential arbitrage profit 
per contract, given this set of data, is closest to 
 
a. USD 10.00 
b. USD 7.50 
c. USD 5.00 
d. USD 1.50 

 
Copyright © 2009 Global Association of Risk Professionals         13    
All rights reserved.
2009 FRM Practice Exams 

30. Research and model projections indicate that a specific event is likely to move the CHF against the 
USD. While the direction of the move is highly uncertain, it is highly likely that magnitude of the 
move will be significant. Based on this information, which of the following strategies would provide 
the largest economic benefit? 
 
a. Long a call option on USD/CHF and short a put option on USD/CHF with the same strike price 
and expiration date 
b.  Long a call option on USD/CHF and long a put option on USD/CHF with the same strike price and 
expiration date 
c.  Short a call option on USD/CHF and long a put option on USD/CHF with the same strike price 
and expiration date 
d. Short a call option on USD/CHF and short a put option on USD/CHF with the same strike price 
and expiration date 
 
 
 
 
 
31. Initially, the call option on Big Kahuna Inc. with 90‐days to maturity trades at USD 1.40. The option 
has a delta of 0.5739. A dealer sells 200 call option contracts and to delta‐hedge the position, the 
dealer purchases 11,478 shares of the stock at the current market price of USD 100 per share. The 
following day, the prices of both the stock and the call option increase. Consequently, delta 
increases to 0.7040. To maintain the delta hedge, the dealer should: 
 
a. Purchase 2,602 shares 
b. Sell 2,602 shares 
c. Purchase 1,493 shares 
d. Sell 1,493 shares 
 
 
 
 
32. Which of the following strategies creates a calendar spread? 
 
a. Sell a call option with a certain strike price and buy a longer maturity call option with the same 
strike price 
b. Buy a call option with a certain strike price and buy a longer maturity call option with the same 
strike price 
c. Sell a call option with a certain strike price and buy a shorter maturity call option with the same 
strike price 
d. Buy a call option with a certain strike price and sell a longer maturity call option with the same 
strike price 
 

 
Copyright © 2009 Global Association of Risk Professionals         14    
All rights reserved.
2009 FRM Practice Exams 

33. Which of the following underlying macro‐economic conditions would leave an emerging market 
most vulnerable to the contagion effects of a currency crisis?  
 
a. Large current account surplus, low foreign exchange reserves, non‐convertible currency 
b. Large current account deficit, low foreign exchange reserves, fully convertible currency 
c. Small current account deficit, high foreign exchange reserves, non‐convertible currency 
d. Large current account surplus, high foreign exchange reserves, fully convertible currency 
 
 
 
 
 
 
34. Consider an FRA (forward rate agreement) with the same maturity and compounding frequency as a 
Eurodollar futures contract. The FRA has a LIBOR underlying. Which of the following statements are 
true about the relationship between the forward rate and the futures rate? 
 
a. They should be exactly the same 
b. The forward rate is normally higher than the futures rate 
c. The forward rate is normally lower than the futures rate 
d. They have no fixed relationship 

 
 

 
35. Your bank is an active player in the commodity market. The view of the economist of the bank is 
that inflation is expected to rise moderately in the near term and market volatility is expected to 
remain low. The traders are advised to undertake deals on the metals exchange to align your book 
to conform with the expectations of the economist of the bank. As risk manager, you are asked to 
monitor the positions of the traders to make sure that they have the exposures to inflation and 
market volatility sought by the bank. Which trader has taken an appropriate position among the 
traders you are monitoring?  
 
a. Trader A bought a call and a put, both with 90‐days to expiration and with strike price equal to 
the existing spot level 
b. Trader B bought a put option with a down‐and‐in knock in feature 
c. Trader C bought a call option at the existing spot levels and sold a call at a higher strike price, 
both with 90‐days to expiration 
d. Trader D sold a call and bought a put at the existing levels, both with 90‐days to expiration 
 
 
 

 
Copyright © 2009 Global Association of Risk Professionals         15    
All rights reserved.
2009 FRM Practice Exams 

36. The information ratio of the Sterole US Fund for 2006 against the S&P 500, its benchmark index, is 1.  
For the same time period, the fund’s Sharpe ratio is 2, the fund has a tracking error of 7% against 
the S&P 500, and the standard deviation of fund returns is 5%. The risk‐ free rate in the US is 4%. 
Calculate the return for the S&P 500 during the time period. 
 
a. 3.5% 
b. 7% 
c. 11% 
d. 14% 
 
 
 
37. A fund manager recently received a report on the performance of his portfolio over the last year. 
According to the report, the portfolio return is 9.3%, with a standard deviation of 13.5%, and a beta 
of 0.83. The risk‐free rate is 3.2%, the semi‐standard deviation σL(Rp) of the portfolio is 8.4%, and the 
tracking error of the portfolio to the benchmark index is 2.8%. What is the difference between the 
value of the fund’s Sortino ratio (computed relative to the risk‐free rate) and its Sharpe ratio? 
 
a. 0.274 
b. 1.727 
c. 0.653 
d. ‐0.378 
 
 
 
38. Which of the following statements about the linear regression of the return of a portfolio over the 
return of its benchmark presented below are correct? 
 
Portfolio parameter                   Value 
Beta                                        1.25 
Alpha                                      0.26 
Coefficient of determination       0.66 
Standard deviation of error       2.42 
 
I. The correlation is 0.71 
II. 34% of the variation in the portfolio return is explained by variation in the benchmark 
return 
III. The portfolio is the dependent variable 
IV. For an estimated portfolio return of 12%, the confidence interval at 95% is [7.16%;16.84%] 
 
a. II and IV 
b. III and IV 
c. I, II and III 
d. II, III and IV  

 
Copyright © 2009 Global Association of Risk Professionals         16    
All rights reserved.
2009 FRM Practice Exams 

39. Your Board of Directors wants a comprehensive review of each business units’ operational risk 
activities.  As the head of the corporate operational risk unit, you know that little has been done to 
implement an operational risk process at the business unit level and that you need to immediately 
come up with a framework.  Which of the following statements offers the best strategy?  
 
I. The audit committee of the Board should first define its objectives to ensure that all the 
firm’s business units’ operational risk programs are providing required information 
II. The auditing department is to be charged with developing an operational risk program for 
each business unit, with the business unit being made clearly aware that they will be held 
accountable for its implementation 
III. That your department immediately assess the operational risk for each business unit using 
independent data feeds to ensure the information fed into the assessment cannot be 
manipulated  
IV. A senior manager from each profit center is to be charged with developing their own 
operational risk self assessment program based on guidelines you provide. 
  
a. I only 
b. I and IV only 
c. I and III only 
d. IV only  
 
 
 
 
40.  Which of the following risk management strategies of a firm which has principal payments to make 
on its debt in one year that substantially exceed the market value of its assets is most likely to be in 
the interest of the shareholders? 
 
a. Reduction of the overall risk level of the firm 
b. Increase of the overall risk level of the firm 
c. Keep the same risk level 
d. It is impossible to say which risk management strategy the shareholders prefer 
 
 
 
 
END OF 2009 FRM Level I PRACTICE EXAM  

 
Copyright © 2009 Global Association of Risk Professionals         17    
All rights reserved.
2009 FRM Practice Exams 

This page is intentionally left blank.  

 
Copyright © 2009 Global Association of Risk Professionals         18    
All rights reserved.
2009 FRM Practice Exams 

2009 FRM Level I Practice Exam  
Answer Key 

1. a. b. c. d. 21. a. b. c. d.
2. a. b. c. d. 22. a. b. c. d.
3. a. b. c. d. 23. a. b. c. d.
4. a. b. c. d. 24. a. b. c. d.
5. a. b. c. d. 25. a. b. c. d.
6. a. b. c. d. 26. a. b. c. d.
7. a. b. c. d. 27. a. b. c. d.
8. a. b. c. d. 28. a. b. c. d.
9. a. b. c. d. 29. a. b. c. d.
10. a. b. c. d. 30. a. b. c. d.
11. a. b. c. d. 31. a. b. c. d.
12. a. b. c. d. 32. a. b. c. d.
13. a. b. c. d. 33. a. b. c. d.
14. a. b. c. d. 34. a. b. c. d.
15. a. b. c. d. 35. a. b. c. d.
16. a. b. c. d. 36. a. b. c. d.
17. a. b. c. d. 37. a. b. c. d.
18. a. b. c. d. 38. a. b. c. d.
19. a. b. c. d. 39. a. b. c. d.
20. a. b. c. d. 40. a. b. c. d.

 
Copyright © 2009 Global Association of Risk Professionals         19    
All rights reserved.
2009 FRM Practice Exams 

This page is intentionally left blank.  

 
Copyright © 2009 Global Association of Risk Professionals         20    
All rights reserved.
2009 FRM Practice Exams 

2009 FRM Level I Practice Exam  
Answers & Explanations 
 
1. To hedge against future, unanticipated, and significant increases in borrowing rates, which of the 
following alternatives offers the greatest flexibility for the borrower? 
   
a. Fixed for floating swap 
b. Interest rate collar 
c. Interest rate floor 
d. Call swaption 
 
CORRECT: D 
 
The question focuses on flexible management of borrowing expenses.  While a fixed for floating 
swap could reduce borrowing expenses, it is a long‐term contractual commitment to exchange 
payments.  If interest rates decline, the borrower may gross up to the agreed fixed rate. An interest 
rate collar is a combination of an interest rate floor and cap, i.e., it locks in the interest expenses 
within a tight range.  Moreover, collars usually offer interest rate protection at one particular point 
of time unless several contracts with different maturities are exchanged. A call swaption gives the 
company the right to enter into a swap when the borrowing expenses exceed a certain reference 
rate.  If the reference rate is below the borrowing expenses, the option is not exercised. 
 
Reference: Hull, Chapter 7. 
 
 
 
 
2. An investment bank uses the Exponentially Weighted Moving Average (EWMA) technique with 
lambda of 0.9 to model the daily volatility of a security. The current estimate of the daily volatility is 
1.5%. The closing price of the security is USD 20 yesterday and USD 18 today. Using continuously‐
compounded returns, what is the updated estimate of the volatility? 
 
a. 5.44% 
b. 3.62% 
c. 2.96% 
d. 1.31% 
 
CORRECT: B 
 
The current return of the security is = ln (18/20) = ‐10.536%. 
 

 
Copyright © 2009 Global Association of Risk Professionals         21    
All rights reserved.
2009 FRM Practice Exams 

Using an EWMA model, the updated volatility is given as:  
V(t)   = {lambda* ((V[t‐1]^2) +(1 – lambda)*(current return^2)} ^ 0.5 
= {0.9     * ((0.015^2) + (1 ‐ 0.9) * ( ‐0.10536^2  )}  ^ 0.5 
= 3.62% 
 
INCORRECT: A – Forgets to square the volatility terms 
INCORRECT: C – Forgets to square the volatility terms and to take the square root of the resulting 
variance, then miscalculates conversion to percentage. 
INCORRECT: D – Forgets to take the square root of the variance, then miscalculates conversion to 
percentage. 
 
Reference : Hull, Chapter 21. 
 
 
 
 
3. Consider two stocks A and B.  Assume their annual returns are jointly normally distributed, the 
marginal distribution of each stock has mean 2% and standard deviation 10%, and the correlation is 
0.9.  What is the expected annual return of stock A if the annual return of stock B is 3%? 
 
a. 2.9% 
b. 2% 
c. 1.1% 
d. 4.7% 
 
CORRECT: A 

E[ra | rb = x] = μa + (ρabσaσb/σ2a)(x – μb) = 0.02 + 0.9 * (0.03 – 0.02) = 0.029

Reference: Damodar Gujarati, Chap 2,3. 
 
 
 
 
4. In pricing a derivative using the Monte Carlo method, we need to simulate a reasonable number of 
paths for the price of the underlying asset.  Suppose we use a simple model for the return of the 
underlying asset: 

y(t) = drift*Δt + vol * √ Δt * e(t), and e(t) is distributed ~ N(0,1), 

where drift and vol are known parameters and Δt is the step size.

 
Copyright © 2009 Global Association of Risk Professionals         22    
All rights reserved.
2009 FRM Practice Exams 

The generation of each path requires a number of steps. Which of the following describes the 
correct procedure? 
 
a. Generate a random number from a normal distribution N(0,1), use the inverse normal function 
to get e(t), which will be fed into the model to get y(t). Repeat the same procedure until you get 
the full desired path. 
b. Generate a random number from a normal distribution N(0,1), use the cumulative normal 
function to get e(t), which will be fed into the model to get y(t). Repeat the same procedure 
until you get the full desired path. 
c. Generate a random number from a uniform distribution defined in [0,1], use the inverse 
cumulative normal function to get e(t), which will be fed into the model to get y(t). Repeat the 
same procedure until you get the full desired path. 
d. Generate a random number from a uniform distribution defined in [0,1], use the cumulative 
normal function to get e(t), which will be fed into the model to get y(t). Repeat the same 
procedure until you get the full desired path. 
 
CORRECT: C 
 
This question wants to test if the candidate knows the basic steps to generate a very simple path: 
answering this question means that the candidate would be able to build a simple spreadsheet 
showing the Monte Carlo logic. 
The correct procedure is the one described in c); while a), b) and d) are nonsensical calculations. 
 
Reference: Philippe Jorion, Value at Risk,The New Benchmark for Managing Financial Risk, 3rd edition 
(New York: McGraw‐Hill, 2007), Chapter 12. 
 
 
 
 
5. A risk manager estimates the daily variance (ht) using a GARCH model on daily returns (rt): 
 
ht = α0 + α1r2t‐1 + βht‐1 
 
Assume the model parameter values are α0 = 0.005, α1 = 0.04, β = 0.94.  The long‐run annualized 
volatility is approximately: 
 
a. 25.00% 
b. 13.54% 
c. 72.72% 
d. 7.94% 
 
CORRECT: D 
 

 
Copyright © 2009 Global Association of Risk Professionals         23    
All rights reserved.
2009 FRM Practice Exams 

The long‐run variance is 0.005/(1‐0.04‐0.94) =0.005/0.02 = 0.25.  The daily vol is thus the square 
root, or 0.5% and annual vol 7.935%. 
 
INCORRECT: A – The daily variance is indeed 0.25%, and the daily volatility 0.5% but this needs to be 
annualized. 
INCORRECT: B – Miscalculates variance as sqrt(0.04/(1 – 0.94 – 0.005)) * 15.87 = 13.54% 
INCORRECT: C – Miscalculates variance as 0.04/(1 – 0.94 – 0.005) = 72.72% 
 
Reference: Hull, Chapter 21. 
 

6. A single stock has a price of USD 10 and a current daily volatility of 2%. Using the delta‐normal 
method, the VaR at the 95% confidence level of a long at‐the‐money call on this stock over a 1‐day 
holding period is approximately: 
 
a. USD 0.23 
b. USD 1.645 
c. USD 0.33 
d. USD 0.16 
 
CORRECT: D 
 
This question requires candidates to know the formula for the delta‐normal VaR approximation, and 
also to know that the delta of an at‐the‐money call is 0.5. 
 
VaR =| Δ | ×1.645 × σ × S = 0.5 × 1.645 × 0.02 × 10 = 0.1645 . 
 
INCORRECT: A – We get A by using 2.326 instead of 1.645. 
INCORRECT: B – We get B if we use 2 instead of 2% for the volatility. 
INCORRECT: C – We get C if we use a delta of 1. 
 
Reference: Allen et al, Chapter 3, 86‐89 
 
 
 
 
7. A portfolio consists of two zero coupon bonds, each with a current value of USD 10. The first bond 
has a modified duration of 1 year and the second has a modified duration of 9 years. The yield curve 
is flat and all yields are 5%. Assume all moves of the yield curve are parallel shifts. Given that the 
daily volatility of the yield is 1%, which of the following is the best estimate of the portfolio daily VaR 
at the 95% confidence level? 
 

 
Copyright © 2009 Global Association of Risk Professionals         24    
All rights reserved.
2009 FRM Practice Exams 

a. USD 2.33 
b. USD 1.65 
c. USD 0.82 
d. USD 1.16 
 
CORRECT: B 
This question assesses candidates’ abilities to apply the duration VaR formula to two bonds 
simultaneously and to recall that the duration of a zero coupon bond is equal to the bond maturity.  
Using an obvious extension of Jorion’s equation 9.5 
 
VaR = D1 × V1 × 1.645 × σ + D2 × V2 × 1.645 × σ = ( D1 × V1 + D2 × V2 ) × 1.645 × σ =  
( D1 + D2 ) × 10 × 1.645 × σ = 10 × 10 × 1.645 × 0.01 = 1.645  
 
INCORRECT: A – The 99% confidence level VaR 
INCORRECT: C – Arises if the candidate mistakenly divides the correct answer by the number of bond  
INCORRECT: D – Makes both mistakes 
 
Reference: Allen et al. 
 
 
8. Consider the following three methods of estimating the P&L of a bullet bond: full repricing, duration 
(PV01), and duration plus convexity.  Ranking the estimated P&L impact of a large negative yield 
shock from the lowest P&L impact to the highest P&L impact, what is the ranking of the methods to 
estimate the P&L impact?  
 
a. Duration plus convexity, duration, full repricing 
b. Full repricing, duration plus convexity, duration 
c. Duration, duration plus convexity, full repricing 
d.  Duration, full repricing, duration plus convexity  
 
CORRECT: C 
 
The price / yield line with yield on the x axis and price on the y axis is convex to the origin.  The 
duration at any yield level is the tangent to that curve.  Therefore, except at the exact point of 
tangency, duration will always underestimate the price change. 
 
INCORRECT: A – Duration will always underestimate price change for negative yield shocks 
INCORRECT: B – Full repricing will never generate a smaller positive price change than duration 
because duration represents the point of tangency 
INCORRECT: D – Full repricing will generate a higher price for a large negative yield change than will 
duration plus convexity 
 
Reference: Allen, Boudoukh, Saunders, Chapter 3 

 
Copyright © 2009 Global Association of Risk Professionals         25    
All rights reserved.
2009 FRM Practice Exams 

9. Consider a position in a 5‐year receive‐fixed swap that makes annual payments on a USD 100 million 
notional.  The floating leg has just been reset.  The term structure is flat at 5%, the Macaulay 
duration of a 5‐year par bond is 4.5 years, and the annual volatility of yield changes is 100bp.  Your 
best estimate of the swap’s VaR with 95% confidence over the next month is 
 
a. USD 1.6 million 
b. USD  2.0 million 
c. USD 5.5 million 
d. USD 7.1 million 
 
CORRECT: A 

Because the floating‐rate leg has just been reset, its duration is 1.  Net duration is 4.5‐1=3.5 year, or 
modified duration of 3.5/1.05=3.33.  The 95% VaR of monthly changes in yields is 1.65*1%/√12 = 
0.48%.  Multiplying, this gives USD 100*0.48%*3.33=USD 1.588 
 
INCORRECT: B – This uses a net duration of 4.5 years and ignores the duration of the floating‐rate 
leg. 
INCORRECT: C – This is the annual VaR, but should be translated to a monthly horizon. 
INCORRECT: D – This is the annual VaR computed by ignoring the duration of the floating‐rate leg. 
 
Reference: Allen et al. 
 
 
10. If the gold lease rate is higher than the risk‐free rate, what is the market structure of the forward 
market for gold? 
 
a. Contango 
b. Backwardation 
c. Inversion 
d. Need more information to determine 
  
 CORRECT: B 
 
A lease rate higher than the risk fee rate will force a negatively sloped forward curve, i.e. 
backwardation 
 
 INCORRECT: A – The forward price = spot*exp( risk free rate ‐ lease rate).  If the lease rate is higher 
than the risk free rate, forwards will be lower than spot, implying contango 
INCORRECT: C – The term inversion is used to describe yield curves, not commodity forwards 
INCORRECT: D – There is enough information in the question to provide an answer 
 
Reference: MacDonald, Chapter 6 

 
Copyright © 2009 Global Association of Risk Professionals         26    
All rights reserved.
2009 FRM Practice Exams 

11. The price of a 3‐year zero coupon government bond is 85.16.  The price of a similar 4‐year bond is 
79.81.  What is the one‐year implied forward rate from year 3 to year 4? 
 
a. 5.4% 
b. 5.5% 
c. 5.8% 
d. 6.7% 

CORRECT: D 
Price of three bond 85.16
1 + Forward rate = = = 1.067034
Price of four year bond 79.81
 
Forward rate = 0.067034 or 6.7%
 
INCORRECT: A – This is B/C 
INCORRECT: B – This is the return of the 3‐year bond  
INCORRECT: C – This is the return of the 4‐year bond  
 
Reference: Tuckman 
 
 
 
12. A portfolio manager has a bond position worth USD 100 million.  The position has a modified 
duration of 8 years and a convexity of 150 years.  Assume that the term structure is flat.  By how 
much does the value of the position change if interest rates increase by 25 basis points? 
 
a. USD ‐1,953,125 
b. USD ‐1,906,250 
c. USD ‐2,046,875 
d. USD ‐2,187,500  
 
CORRECT:  A 
ΔV = - D mod × Δy × V + 0.5 × Convexity × Δy 2 × V
ΔV = - 8 × 0.0025 × 100M + 0.5 × 150 × (0.0025) 2 × 100M
 
ΔV = - 2M + 46,875
ΔV = - 1,953,125
 
INCORRECT: B – Omits 0.5 from the second term 
INCORRECT: C – Subtracts the second term 
INCORRECT: D – Makes both mistakes 
 
Reference: Tuckman 

 
Copyright © 2009 Global Association of Risk Professionals         27    
All rights reserved.
2009 FRM Practice Exams 

13. A firm is going to buy 10,000 barrels of West Texas Crude Oil.  It plans to hedge the purchase using 
the Brent Crude futures contract.  The correlation between the spot and futures prices is 0.72.  The 
volatility of the spot price is 0.35 per year.  The volatility of the Brent Crude futures price is 0.27 per 
year.  What is the hedge ratio for the firm? 
 
a. 0.5554 
b. 0.9333 
c. 1.2099 
d. 0.8198 
 
CORRECT: B 
⎛ 0.35 ⎞
N = 0.72 × ⎜ ⎟
⎝ 0.27 ⎠  
N= 0.9333
 
INCORRECT: A – Inverts the spot volatility and the futures volatility 
INCORRECT: C – Uses variances 
INCORRECT: D – Uses square roots of the volatilities 
 
Reference: Hull, Chapter 3 
 
 
 
 
14. It is June 2nd and a fund manager with USD 10 million invested in government bonds is concerned 
that interest rates will be highly volatile over the next three months.  The manager decides to use 
the September Treasury bond futures contract to hedge the value of the portfolio.  The current 
futures price is 95.0625.  Each contract is for the delivery of USD 100,000 face value of bonds.  The 
duration of the manager’s bond portfolio in three months will be 7.8 years.  The cheapest to deliver 
bond in the Treasury bond futures contract is expected to have a duration of 8.4 years at maturity of 
the contract.  At the maturity of the Treasury bond futures contract, the duration of the underlying 
benchmark Treasury bond is 9 years. What position should the fund manager undertake to mitigate 
his interest rate risk exposure? 
 
a. Short 94 contracts 
b. Short 98 contracts 
c. Short 105 contracts 
d. Short 113 contracts 
 
CORRECT:  B.   

 
Copyright © 2009 Global Association of Risk Professionals         28    
All rights reserved.
2009 FRM Practice Exams 

⎛ 10,000,000 ⎞ ⎛ 7.8 ⎞
N=⎜ ⎟×⎜ ⎟
⎝ 95,062.50 ⎠ ⎝ 8.4 ⎠
 
N = 97.68 or 98 contracts
 
INCORRECT: A – This is made up.   
INCORRECT: C – This leaves out the durations  
INCORRECT: D – This inverts the durations 
 
Reference: Tuckman 
 

 
 

15. A bond trader has bought a position in Treasury Bonds with a 4% annual coupon rate on February 
15, 2015. The DV01 of the position is USD 80,000. The trader decides to hedge his interest rate risk 
with the 4.5% coupon rate Treasury Bonds maturing on May 15, 2017 which has a DV01 of .076 per 
USD 100 face value. To implement this hedge, approximately what face amount of the 4.5% 
Treasury bonds maturing on May 15, 2017 should the trader sell? 
 
a. USD 80,000 
b. USD 10,500,000 
c. USD 80,000,000 
d. USD 105,000,000 
 
CORRECT: D 
 
USD 105,000,000x.076/100 = USD 79,800, which is pretty close to the desired DV01 of USD 80,000. 
To solve for the hedge, solve for F in the equation USD 80,000 = Fx.076/100, giving F = 105,263,158 
 
INCORRECT: A – Selling this amount would offset a DV01 of only USD 80,000x.076/100 = USD 61 
INCORRECT: B – USD 10,500,000x.076/100 = USD 7,980 
INCORRECT: C – USD 80,000,000x.076/100 = USD 60,800 
 
Reference: Tuckman, Chapter 5 
 
 
 
 

 
Copyright © 2009 Global Association of Risk Professionals         29    
All rights reserved.
2009 FRM Practice Exams 

16. Suppose that A and B are random variables, each follows a standard normal distribution, and the 
covariance between A and B is 0.35. What is the variance of (3A + 2B)? 
 
a. 15.10 
b. 14.47 
c. 9.20 
d.  17.20  
 
CORRECT: D 
 
 Since each variable is standardized, its variance is 1. Therefore V(3A+2B) = 9 V(A) + 4 V(B) + 2 x 3 x2 
x Cov(A,B) = 9+4+4.2 = 17.2 
 
INCORRECT: A –   9 + 4 + 6 * 0.35 = 15.1 
INCORRECT: B –   9+ 4 + 12 * 0.35^2 =  
INCORRECT: C –   3 + 2 + 12 * 0.35 = 9.2 
 
Reference: Damodar Gujarati 
 
 
 
17. Consider a stock price S that follows a geometric Brownian motion dS = μ S dt + β S dz, with β
strictly positive and μ a fixed value.  Which of the following statements is true? 
 
a. If the drift μ is negative, the price one year from now will be below today’s price. 
b. The instantaneous rate of return on the stock follows a uniform distribution. 
c. The stock price S follows a lognormal distribution. 
d. This model imposes mean reversion. 
 
CORRECT: C 
 
INCORRECT: A – The expected price is less than today’s price, but not the price in all the states of 
world. 
INCORRECT: B – The instantaneous rate of return on the stock follows normal distribution.  
INCORRECT: D – This model does not impose mean reversion. 
 
rd 
Reference:  Philippe Jorion, Value at Risk: The New Benchmark for Managing Financial Risk, 3 ed. 
(New York: McGraw‐Hill, 2007). Chapter 12 
 

 
Copyright © 2009 Global Association of Risk Professionals         30    
All rights reserved.
2009 FRM Practice Exams 

18. The joint probability distribution of random variables X and Y is given by f(x,y) = kxy for x = 1, 2, 3, y = 
1, 2, 3, and k is a positive constant. What is the probability that X + Y will exceed 5? 
 
a. 1/9 
b. 1/4 
c. 1/36 
d. Cannot be determined 
 
CORRECT: B 
3 3
Note that ∑∑ f ( x, y) = 1
x =1 y =1

Substituting the various values of x and y, we get f(1,1)=k, f(1,2)=2k, f(1,3)=3k, f(2,1)=2k, f(2,2)=4k, 
f(2,3)=6k, f(3,1)=3k, f(3,2)=6k, and f(3,3)=9k. Therefore, 
   k1 + 2k + 3k + 2k + 4k + 6k + 3k + 6k + 9k = 1 
so that, 36k = 1 and k=1/36. 
P(X+Y>5)= f(3,3)= 1/36 x 3 x 3 = 1/4 
 
Reference: Damodar Gujarati 
 
 
 
19. Which of the following statements regarding Hypothesis Testing is incorrect? 
 
a.  Hypothesis testing is used to make inferences about the parameters of a given population on 
the basis of statistics computed for a sample that is drawn from that population. 
b. Type II error refers to the failure to reject the null hypothesis when it is actually false. 
c. The p‐value decision rule is to reject the null hypothesis if the p‐value is greater than the 
significance level. 
d.  All else being equal, the decrease in the chance of making a Type I error comes at the cost of 
increasing the probability of making a Type II error. 
 
CORRECT: C 
 
 The true statement is to reject Ho if the p‐value is smaller than the significance level. 
 
INCORRECT: A – Statement A is correct regarding the primary use of Hypothesis Testing. 
INCORRECT: B – Statement B is correct regarding the definition of type II error. 
INCORRECT: D – Statement D is correct because type I error and type II error are in tradeoff. 
 
Reference: Damodar Gujarati 

 
Copyright © 2009 Global Association of Risk Professionals         31    
All rights reserved.
2009 FRM Practice Exams 

20. If stock returns are independently identically normally distributed and the annual volatility is 30%, 
then the daily VaR at the 99% confidence level of a stock market portfolio is approximately: 
 
a. 2.41% 
b. 3.11% 
c. 4.40% 
d. 1.89% 
 
CORRECT: C 
 
The 1‐day volatility is s * (1/252)^0.5 = 0.3 * 0.629941 = 0.018898. The VaR at the 99% confidence 
level is then equal to 2.32635 * 0.018898 = 4.40% 
 
INCORRECT: A – One gets A if one uses 1.645 instead of 2.326 
INCORRECT: B – One gets B if one uses the monthly volatility instead of the daily one 
INCORRECT: D – One gets D is the daily volatility 
 
Reference: Allen, Boudoukh and Saunders, 2004, chapter 1, p 6‐8 
 
 
 
 
21. The current value of the S&P 500 index is 1457, and each S&P futures contract is for delivery of USD 
250 times the index.  A long‐only equity portfolio with market value of USD 300,100,000 has beta of 
1.1.  To reduce the portfolio beta to 0.75, how many S&P futures contract should you sell? 
 
a. 618 contracts 
b. 288 contracts 
c. 574 contracts 
d. 906 contracts 
 
CORRECT: B 
 
No of contracts = [0.75 – 1.1)/ 1]* [300,100,000 / {250 * 1,457}] = ‐288.36 Æ sell 288 contracts 
 
INCORRECT: A –   ‐617.9135209 = ‐1*(0.75)* (300100000 / (250*1457)) 
INCORRECT: C –   ‐561.74 = ‐1(0.75/1.1)* (300100000 / (250*1457)) 
INCORRECT: D –   ‐906.273164 = ‐1* (1.1)* (300100000 / (250*1457)) 
 
Reference: Hull, Options, Futures and Other Derivatives, Chapter 3 and 4; Anthony Saunders, 
Financial Institutions Management, Chapter 10 

 
Copyright © 2009 Global Association of Risk Professionals         32    
All rights reserved.
2009 FRM Practice Exams 

The following information should be used for the next two questions. 
On January 1, a risk manager observes that the 1‐year continuously compounded interest rate is 5% 
and storage costs of a commodity product A is USD 0.05 per quarter (payable at each quarter end).  
He further observes the following forward prices for product A: 
 
March      5.35 
June      5.90 
September    5.30 
December     5.22 
 
22. Given the following explanation of supply and demand for commodity product A how would you 
best describe its forward price curve from June to December? 
 
  Market description        Explanation 
a. Backwardation          Excess demand for A in early summer 
b. Backwardation          Supply is expected to decline after summer 
c. Contango           Excess demand for A in early summer 
d. Contango          Supply is expected to decline after summer 
 
CORRECT: A 
 
A is correct ‐ when further‐term commodity forwards have lower price than near‐term forwards, the 
market is said to be in ‘backwardation’.  Possible explanation can be seasonality of product A – 
excess demand in early summer causes June forwards to have higher price 
 
INCORRECT: B – Market description is correct, but explanation is not – expected decline in supply 
should increase further‐term commodity forward price 
INCORRECT: C – Wrong market description of contango 
INCORRECT: D – Wrong market description of contango 
 
Reference: Robert L McDonald, Derivatives Markets, Chapter 6 
 
 
23. What is the annualized rate of return earned on a cash‐and‐carry trade entered into in March and 
closed out in June? 
 
a. 8.9% 
b. 9.8% 
c. 35.7% 
d. 39.1% 
 
CORRECT: C 

 
Copyright © 2009 Global Association of Risk Professionals         33    
All rights reserved.
2009 FRM Practice Exams 

By formula F0,T = S0erT + C, where F0,T = June forward price, S0 = March forward price, r = risk 


free interest rate, T = length of cash‐and‐carry, C = storage cost 
Solving 5.90 = 5.35er*3/12 + 0.05 
Solution is r = 35.7% 

INCORRECT: A –   8.9 = LN((5.9‐0.05)/5.35) (forgets to annualize the return) 
INCORRECT: B –   9.8= LN((5.9)/5.35) (forgets to include the storage cost and to annualize the return) 
INCORRECT: D –   39.1= (12/3)LN((5.9)/5.35) – 0.05 (forgets to include the storage cost) 
 
Reference: Robert L McDonald, Derivatives Markets, Chapter 6 

 
 

 
24. An investor sells a June 2008 call of ABC Limited with a strike price of USD 45 for USD 3 and buys a 
June 2008 call of ABC Limited with a strike price of USD 40 for USD 5. What is the name of this 
strategy and the maximum profit and loss the investor could incur? 
 
a. Bear Spread, Maximum Loss USD 2, Maximum Profit USD 3 
b. Bull Spread, Maximum Loss Unlimited, Maximum Profit USD 3 
c. Bear Spread, Maximum Loss USD 2, Maximum Profit Unlimited 
d. Bull Spread, Maximum Loss USD 2, Maximum Profit USD 3 
 
CORRECT: D 
 
Buying a call option at lower stock price and selling call option at higher strike price is called as Bull 
Spread. Bear Spread is buying the call option at higher price and selling the call at lower strike price. 
The Cost of strategy will be USD 3‐USD 5 = ‐USD 2 
The Payoff, when Stock price ST ≤ USD 40 will be ‐USD 2 (the cost of strategy) as none of the option 
will be exercised. 
The Payoff, when stock price ST ≥ 45, (as both options will be exercise) will be USD 5, 
Since the cost of strategy is ‐USD 3, hence profit will be USD 5‐USD 2 = USD 3 
When Stock price is USD 40< ST > USD 45, Only the call option bought by the investor would be 
exercised hence the pay off will be ST – 40, since the cost of strategy is ‐USD 3, The Net profit will be 
ST – 43, which would always be lower than USD 3. 
 
Reference: Hull. Chapter 10‐ Trading Strategies Involving Options.  
 

 
Copyright © 2009 Global Association of Risk Professionals         34    
All rights reserved.
2009 FRM Practice Exams 

25. Which of the following problems are NOT inherent disadvantages of the historical simulation 
approach to estimating VaR? 
 
I.             It gives too little weight to more recent observations 
II. For long‐only portfolios, it is likely to understate VaR following a recent structural increase 
in volatilities 
III. It always ignores the fat tails present in the distribution of returns on many financial 
assets 
IV. Because of the delta approximation, it inadequately measures the risk of nonlinear 
instruments 
 
a. I and II only 
b. II only 
c. I, III and IV only 
d. III and IV only 

CORRECT: C 
 
The disadvantage with the Historical Simulation Model is that it may not recognize the changes in 
volatility and correlation following recent structural changes. The model can be adjusted so that it 
gives more weight to recent observations. The other options, i.e. III & IV, are disadvantages of Monte 
Carlo method and Delta‐normal method.  
 
Reference: Allen et al. 
 
 
 
 
26. A bank holds USD 60 million worth of 10‐year 6.5% coupon bonds that are trading at a clean price of 
101.82. The bank is worried by the exposure due to these bonds but cannot unwind the position for 
fear of upsetting the client. Therefore, it purchases a total return swap (TRS) in which it receives 
annual Libor + 100 bps in return for the mark‐to‐market return on the bond.  For the first year, the 
Libor sets at 6.25% and by the end of the year the clean price of the bonds is at 99.35. The net 
receipt/payment for the bank in the total return swap will be: 
 
a. Receive USD 2.23 million.  
b. Receive USD 1.93 million. 
c. Pay USD 1.93 million. 
d. Pay USD 2.23 million. 
 
CORRECT: B 
 

 
Copyright © 2009 Global Association of Risk Professionals         35    
All rights reserved.
2009 FRM Practice Exams 

It’s the result of this calculation: the notional amount is 60 million USD . Therefore the bank will 
receive the interest payment linked to the LIBOR rate: 60 million USD  * (6,25%+100 bp) = 4. 35 
million USD . 
The bank will pay the fixed coupon plus the change in the value of the bond: 60 million USD  * 6.5% + 
60 million *(99.35%‐101.82%) = 2.418 million USD . 
Hence the total net amount the bank will receive is: 4.35 million USD  ‐ 2.418 million USD  = 1.932 
million USD . 
 
Reference: Hull, Chapter 7  
 
 
 
27. Which of the following trade(s) contain basis risk? 
 
I.            Long 1,000 lots Nov 07 ICE Brent Oil contracts and short 1,000 lots Nov 07 NYMEX WTI 
Crude Oil contracts 
II. Long 1,000 lots Nov 07 ICE Brent Oil contracts and long 2,000 lots Nov 07 ICE Brent Oil at‐
the‐money put 
III. Long 1,000 lots Nov 07 ICE Brent Oil contracts and short 1,000 lots Dec 07 ICE Brent Oil 
contracts 
IV. Long 1,000 lots Nov 07 ICE Brent Oil contracts and short 1,000 lots Dec 07 NYMEX WTI 
Crude Oil contracts 
 
a. I & III 
b. II & IV 
c. IIII & IV 
d. I, III & IV 
 
CORRECT: D 
 
Basis Risk is spread risk, which arise from trading the spread (long and short 2 positively correlated 
assets or same asset with different expiration) 
 
i is spread trade in highly correlated asset with same expiration month 
ii faces with gamma and vega risk 
iii is spread trade in trading the flattening of the forward curve 
iv is spread trade in trading 2 assets with different expiration date 
 
Reference: Robert L. McDonald, Derivatives Markets (Boston: Addison‐Wesley, 2003), Chapter 6. 
 
 
 

 
Copyright © 2009 Global Association of Risk Professionals         36    
All rights reserved.
2009 FRM Practice Exams 

28. According to put‐call parity, buying a put option on a stock is equivalent to: 
 
a. Buying a call option and buying the stock with funds borrowed at the risk‐free rate. 
b. Selling a call option and buying the stock with funds borrowed at the risk‐free rate. 
c. Buying a call option, selling the stock and investing the proceeds  at the risk‐free rate. 
d. Selling a call option, selling the stock and investing the proceeds at the risk‐free rate. 
   
CORRECT: C 
 
Buying a call option, selling the stock and investing the proceeds at the risk‐free rate. Put‐call parity  
states  P = C ‐ S + X e‐RT 
 
INCORRECT: A – Buying a call option is correct, but the rest of the statement is incorrect.  
INCORRECT: B – The entire statement is incorrect. 
INCORRECT: D – Selling a call option is incorrect, but the rest of the statement is correct.  
 
Reference: Hull, Chapter 10 
 
 
 
 
29. A 3 month futures contract on an equity index is currently priced at USD 1000, the underlying index 
stocks are valued at USD 990 and pay dividends at a continuously‐compounded rate of 2 percent 
and the current continuously compounded risk‐free rate is 4 percent. The potential arbitrage profit 
per contract, given this set of data, is closest to 
 
a. USD 10.00 
b. USD 7.50 
c. USD 5.00 
d. USD 1.50 
 
CORRECT: C  
 
According to the fundamental pricing relationship between spot assets and the associated futures, 
the futures price, to prevent arbitrage, should equal 990 x e (0.04 – 0.02) x 0.25 or 995. Hence, the 
futures contract is overvalued, indicating it should be sold and the index should be purchased for an 
arbitrage profit of USD 1000 ‐ USD 995 = USD 5 
 
Reference:  Hull, Chapters 2,3 6. 
 
 
 
 
 

 
Copyright © 2009 Global Association of Risk Professionals         37    
All rights reserved.
2009 FRM Practice Exams 

30. Research and model projections indicate that a specific event is likely to move the CHF against the 
USD. While the direction of the move is highly uncertain, it is highly likely that magnitude of the 
move will be significant. Based on this information, which of the following strategies would provide 
the largest economic benefit? 
 
a. Long a call option on USD/CHF and short a put option on USD/CHF with the same strike price 
and expiration date 
b.  Long a call option on USD/CHF and long a put option on USD/CHF with the same strike price and 
expiration date 
c.  Short a call option on USD/CHF and long a put option on USD/CHF with the same strike price 
and expiration date 
d. Short a call option on USD/CHF and short a put option on USD/CHF with the same strike price 
and expiration date 
 
CORRECT: B 
 
The question tests on understanding of a “straddle” strategy and its application on currency trading. 
A long straddle strategy involves buying (long) a call and put option with the same strike price and 
expiration date, and will benefit most when the underlying moves away from the current equlibrium.  
 
INCORRRECT: A – It sells a put option while it should buy one put 
INCORRECT: C – It sells a call option while it should buy one call 
INCORRECT: D – It sells both the call and put option while it should buy both 
 
Reference: Hull, Chapter 10. 
 
 
 
 
31. Initially, the call option on Big Kahuna Inc. with 90‐days to maturity trades at USD 1.40. The option 
has a delta of 0.5739. A dealer sells 200 call option contracts and to delta‐hedge the position, the 
dealer purchases 11,478 shares of the stock at the current market price of USD 100 per share. The 
following day, the prices of both the stock and the call option increase. Consequently, delta 
increases to 0.7040. To maintain the delta hedge, the dealer should: 
 
a. Purchase 2,602 shares. 
b. Sell 2,602 shares. 
c. Purchase 1,493 shares. 
d. Sell 1,493 shares. 
 
CORRECT: A 
 
Number of calls = 200 contracts x 100 = 20,000 calls. 
 

 
Copyright © 2009 Global Association of Risk Professionals         38    
All rights reserved.
2009 FRM Practice Exams 

Number of shares   = (Number of calls) x (New delta – Old delta) 
      = 20,000 x (0.7040 – 0.5739) 
      = +2,602 shares 
Positive sign indicates that the manager should purchase new shares. 
 
INCORRECT: B – The formula is incorrect, i.e. old delta minus new delta 
INCORRECT: C – The number of shares (instead of number of calls) is used in the calculation 
INCORRECT: D – As per explanation in ‘C’ above and sign error 
 
Reference: Hull Chapters 9 and 10 
 
 
 
32. Which of the following strategies creates a calendar spread? 
 
a. Sell a call option with a certain strike price and buy a longer maturity call option with the same 
strike price 
b. Buy a call option with a certain strike price and buy a longer maturity call option with the same 
strike price 
c. Sell a call option with a certain strike price and buy a shorter maturity call option with the same 
strike price 
d. Buy a call option with a certain strike price and sell a longer maturity call option with the same 
strike price 
 
CORRECT: A 
 
INCORRECT: B – As buy a call option 
INCORRECT: C – As buy a shorter‐maturity call option 
INCORRECT: D – As this is a reverse calendar spread 
 
Reference: John Hull, Chapter 10.  
 
 
 
33. Which of the following underlying macro‐economic conditions would leave an emerging market 
most vulnerable to the contagion effects of a currency crisis?  
 
a. Large current account surplus, low foreign exchange reserves, non‐convertible currency 
b. Large current account deficit, low foreign exchange reserves, fully convertible currency 
c. Small current account deficit, high foreign exchange reserves, non‐convertible currency 
d. Large current account surplus, high foreign exchange reserves, fully convertible currency 
 
CORRECT: B 
 

 
Copyright © 2009 Global Association of Risk Professionals         39    
All rights reserved.
2009 FRM Practice Exams 

INCORRECT: A – Large current account surplus and non‐convertible currency would protect the local 
currency 
INCORRECT: C – High foreign exchange reserves and non‐convertible currency would protect the local 
currency 
INCORRECT: D – Large current account surplus and high foreign exchange would protect the local 
currency 
 
Reference: Saunders, Chapter 15, Foreign Exchange Risk 
 
 
34. Consider an FRA (forward rate agreement) with the same maturity and compounding frequency as a 
Eurodollar futures contract. The FRA has a LIBOR underlying. Which of the following statements are 
true about the relationship between the forward rate and the futures rate? 
 
a. They should be exactly the same 
b. The forward rate is normally higher than the futures rate 
c. The forward rate is normally lower than the futures rate 
d. They have no fixed relationship 
 
CORRECT: C  
 
As Eurodollar futures contract is marked to market and settled daily, normally forward rate is 
adjusted lower, so called convexity adjustment, by: 
1 2
Forward rate = Futures rate – σ T1T2
2

Reference: Hull, Chapter 6. 
 
 
35. Your bank is an active player in the commodity market. The view of the economist of the bank is 
that inflation is expected to rise moderately in the near term and market volatility is expected to 
remain low. The traders are advised to undertake deals on the metals exchange to align your book 
to conform with the expectations of the economist of the bank. As risk manager, you are asked to 
monitor the positions of the traders to make sure that they have the exposures to inflation and 
market volatility sought by the bank. Which trader has taken an appropriate position among the 
traders you are monitoring?  
 
a. Trader A bought a call and a put, both with 90‐days to expiration and with strike price equal to 
the existing spot level 
b. Trader B bought a put option with a down‐and‐in knock in feature 
c. Trader C bought a call option at the existing spot levels and sold a call at a higher strike price, 
both with 90‐days to expiration 
d. Trader D sold a call and bought a put at the existing levels, both with 90‐days to expiration 

 
Copyright © 2009 Global Association of Risk Professionals         40    
All rights reserved.
2009 FRM Practice Exams 

 
CORRECT: C 
 
C Is correct, as the strategy popularly known as the bull spread will result in positive payoff when the 
spot rises. As inflation increases, spot levels in commodities are expected to rise. Selling a call at 
higher level will reduce the cost of the strategy. Although it may limit the upside, but that would be 
in line with the view as only a moderate rise is expected in spot. 
 
INCORRECT: A – Is incorrect, as the strategy popularly known as a straddle is to be used when the 
view is that the volatility in the market will rise, and there is no directional view on the spot 
INCORRECT: B – Is incorrect, as the above option will be suitable when the spot is expected to fall 
from the existing levels 
INCORRECT: D – Is incorrect, as the payoff in this case is similar to short position in spot and would 
make sense when the underlying is expected to fall 
 
Reference: Hull, Chapter 10. 
 
 
 
36. The information ratio of the Sterole US Fund for 2006 against the S&P 500, its benchmark index, is 1.  
For the same time period, the fund’s Sharpe ratio is 2, the fund has a tracking error of 7% against 
the S&P 500, and the standard deviation of fund returns is 5%. The risk‐ free rate in the US is 4%. 
Calculate the return for the S&P 500 during the time period. 
 
a. 3.5% 
b. 7% 
c. 11% 
d. 14% 
 
CORRECT: B 
 
Sharpe Ratio = 2 
(Fund Return – Risk Free Rate)/SD = 2 
(Fund Return – 4%)/5% = 2 
Fund Return = 14% 
 
Information Ratio = 1 
(Fund Return – S&P 500 Return)/ Tracking Error = 1 
(14% ‐ S&P 500 Return) / 7% = 1 
S&P 500 Return = 7% 
 
INCORRECT: A – Incorrectly divides S&P 500 Return by 2 
INCORRECT: C – The candidate might use the Tracking Error as the Numerator in both the Ratios 

 
Copyright © 2009 Global Association of Risk Professionals         41    
All rights reserved.
2009 FRM Practice Exams 

Sharpe Ratio = 2 
(Fund Return – Risk Free Rate)/Tracking Error = 2 
(Fund Return – 4%)/7% = 2 
Fund Return = 18% 
Information Ratio = 1 
(Fund Return – S&P 500 Return)/ Tracking Error = 1 
(18% ‐ S&P 500 Return) / 7% = 1 
S&P 500 Return = 11% 
INCORRECT: D – The candidate can stop with the fund return calculation, and end up with 14% 
Sharpe Ratio = 2 
(Fund Return – Risk Free Rate)/SD = 2 
(Fund Return – 4%)/5% = 2 
Fund Return = 14% 
 
Reference: Amenc and Le Sourd, Portfolio Theory and Performance Analysis. Chapter 4 
 
 

37. A fund manager recently received a report on the performance of his portfolio over the last year. 
According to the report, the portfolio return is 9.3%, with a standard deviation of 13.5%, and a beta 
of 0.83. The risk‐free rate is 3.2%, the semi‐standard deviation σL(Rp) of the portfolio is 8.4%, and the 
tracking error of the portfolio to the benchmark index is 2.8%. What is the difference between the 
value of the fund’s Sortino ratio (computed relative to the risk‐free rate) and its Sharpe ratio? 
 
a. 0.274 
b. 1.727 
c. 0.653 
d. ‐0.378 
 
CORRECT: A  
R p − RF 9 .3 % − 3 .2 %
Sharpe ratio equals to = = 0.452
σ (R p ) 13.5%
R p − RF 9.3% − 3.2%
While Sortino ratio equals to = = 0.726
σ L ( RP ) 8.4%
Tracking error is used to calculate the value of the information ratio, which is defined as 
R p − RB
, The calculation of information ratio is not required in this question.  
σ ( RP − RB )
0.726 – 0.452 = 0.274 
 
INCORRECT: B –    2.178 – 0.452 = 1.727 

 
Copyright © 2009 Global Association of Risk Professionals         42    
All rights reserved.
2009 FRM Practice Exams 

INCORRECT: C –    0.726 – 0.0.73 = 0.653  (0.073 = (.093 – 0.032)/0.83 
INCORRECT: D –   0.73‐0.452=0.378 
 
Reference: Amenc and Le Sourd, Portfolio Theory and Performance Analysis. Chapter 4 
 
 

 
 

38. Which of the following statements about the linear regression of the return of a portfolio over the 
return of its benchmark presented below are correct? 
 
Portfolio parameter                   Value 
Beta                                          1.25 
Alpha                                        0.26 
Coefficient of determination       0.66 
Standard deviation of error       2.42 
 
I.   The correlation is 0.71 
II.   34% of the variation in the portfolio return is explained by variation in the benchmark return 
III.   The portfolio is the dependent variable 
IV.   For an estimated portfolio return of 12%, the confidence interval at 95% is [7.16%;16.84%] 
 
a. II and IV 
b. III and IV 
c. I, II and III 
d. II, III and IV  
 
CORRECT: B  
 
The portfolio return is the dependent variable and for an estimated portfolio return of 12%, the 95% 
confidence interval is [12% ‐ 2 * 2.42%, 12% + 2 * 2.42%] or [7.16%, 16.84%]. 
 
However, the correlation is the square root of the coefficient of determination and is therefore equal 
to 0.81, and 66% of the variation in the portfolio returns is explained by variation in the benchmark 
return. 
 
Reference: Amenc and Le Sourd, Portfolio Theory and Performance Analysis. Chapter 4 
 
 
 

 
Copyright © 2009 Global Association of Risk Professionals         43    
All rights reserved.
2009 FRM Practice Exams 

39. Your Board of Directors wants a comprehensive review of each business units’ operational risk 
activities.  As the head of the corporate operational risk unit, you know that little has been done to 
implement an operational risk process at the business unit level and that you need to immediately        
come up with a framework.  Which of the following statements offers the best strategy? 
 
I.            The audit committee of the Board should first define its objectives to ensure that all the 
firm’s business units’ operational risk programs are providing required information 
II. The auditing department is to be charged with developing an operational risk program for 
each business unit, with the business unit being made clearly aware that they will be held 
accountable for its implementation 
III. That your department immediately assess the operational risk for each business unit 
using independent data feeds to ensure the information fed into the assessment cannot 
be manipulated  
IV. A senior manager from each profit center is to be charged with developing their own 
operational risk self assessment program based on guidelines you provide. 
 
a. I only 
b. I and IV only 
c. I and III only 
d. IV only  
 
CORRECT: D 
 
The best strategy for developing an operational risk framework is to empower business units with 
the responsibility, accountability and authority to manage their own operational risks.  The business 
units know their risks the best.   
 
INCORRECT: A – ‘I’ is not the responsibility of the Audit Committee of the Board 
INCORRECT: B – The auditing department is not the best assessor of an individual business unit’s risk, 
in fact many audit staff do not fully understand the risks of many of a firm’s activities 
INCORRECT: C – III’ is duplicative and should not come from the corporate department 
 
Reference:  Risk Management and Capital Adequacy, Gallati, 2003. 
 

40.  Which of the following risk management strategies of a firm which has principal payments to make 
on its debt in one year that substantially exceed the market value of its assets is most likely to be in 
the interest of the shareholders? 
 
a. Reduction of the overall risk level of the firm 
b. Increase of the overall risk level of the firm 
c. Keep the same risk level 
d. It is impossible to say which risk management strategy the shareholders prefer 

 
Copyright © 2009 Global Association of Risk Professionals         44    
All rights reserved.
2009 FRM Practice Exams 

 
CORRECT: B 
 
Once a firm is in distress, it is not in the interests of shareholders to reduce risk.  If the firm stays in 
distress and eventually defaults, shareholders will end up with worthless shares. In these 
circumstances, management intent on maximizing shareholder value will seek out new risks.  
 
Reference: Risk Management and Derivatives, Stulz, 2003 
 
 
 
END OF 2009 FRM Level I PRACTICE EXAM 
Questions & Explanations  

 
Copyright © 2009 Global Association of Risk Professionals         45    
All rights reserved.
2009 FRM Practice Exams 

This page is intentionally left blank.  

 
Copyright © 2009 Global Association of Risk Professionals         46    
All rights reserved.
2009 FRM Practice Exams 

2009 FRM Full Exam Practice Exam I 
Candidate Answer Sheet 
 
1. a. b. c. d. 26. a. b. c. d.
2. a. b. c. d. 27. a. b. c. d.
3. a. b. c. d. 28. a. b. c. d.
4. a. b. c. d. 29. a. b. c. d.
5. a. b. c. d. 30. a. b. c. d.
6. a. b. c. d. 31. a. b. c. d.
7. a. b. c. d. 32. a. b. c. d.
8. a. b. c. d. 33. a. b. c. d.
9. a. b. c. d. 34. a. b. c. d.
10. a. b. c. d. 35. a. b. c. d.
11. a. b. c. d. 36. a. b. c. d.
12. a. b. c. d. 37. a. b. c. d.
13. a. b. c. d. 38. a. b. c. d.
14. a. b. c. d. 39. a. b. c. d.
15. a. b. c. d. 40. a. b. c. d.
16. a. b. c. d. 41. a. b. c. d.
17. a. b. c. d. 42. a. b. c. d.
18. a. b. c. d. 43. a. b. c. d.
19. a. b. c. d. 44. a. b. c. d.
20. a. b. c. d. 45. a. b. c. d.
21. a. b. c. d. 46. a. b. c. d.
22. a. b. c. d. 47. a. b. c. d.
23. a. b. c. d. 48. a. b. c. d.
24. a. b. c. d. 49. a. b. c. d.
25. a. b. c. d. 50. a. b. c. d.

 
Copyright © 2009 Global Association of Risk Professionals         47    
All rights reserved.
2009 FRM Practice Exams 

 
 
 
 
 
 
 
 
 
 
 
 
This page is intentionally left blank.

 
Copyright © 2009 Global Association of Risk Professionals         48    
All rights reserved.
2009 FRM Practice Exams 

2009 FRM Full Exam Practice Exam I 
Questions 
 
 
1. Given the information provided in the table below, what is the portfolio VaR, at the 99% confidence 
level, of the following 100 million CHF equally weighted investment portfolio?  
 
Expected 
Asset  Volatility Correlation 
Return 
      Stocks  Bonds 
Stocks  24.00%  18%  1   
Bonds  15.00%  6%  0.1  1 
 
 
a. 27.96 million CHF 
b. 22.77 million CHF 
c. 20.97 million CHF 
d. 13.98 million CHF 
 
 
 
 
2. You are asked by your boss to estimate the exposure of a hedge fund to the S&P 500. Though the 
fund claims to mark to market weekly, it does not do so and marks to market once a month. The 
fund also does not tell investors that it simply holds an ETF which is indexed to the S&P500. Because 
of the claims of the hedge fund, you decide to estimate the market exposure by regressing weekly 
returns of the fund on the weekly return of the S&P500. Which of the following properties correctly 
describes a property of your regression estimates?  
 
a. The beta of your regression will be one because the fund holds the S&P 500. 
b. The beta of your regression will be zero because the fund returns are not synchronous with the 
S&P 500 returns. 
c. The intercept of your regression will be positive, showing that the fund has a positive alpha 
when estimated using an OLS regression. 
d. The beta will be misestimated because hedge fund exposures are non‐linear. 

 
 

 
Copyright © 2009 Global Association of Risk Professionals         49    
All rights reserved.
2009 FRM Practice Exams 

3. The following table shows the composition of the GARP Bond Fund. What are the portfolio duration 
and portfolio yield of the fund? 
 
GARP Bond Fund
Rating Amount Duration
Mn USD in years
AAA
Company A 600 1.5
Company B 300 4
Company C 200 2.5
AA
Company D 400 4
Company E 350 0.5
A
Company F 150 1.5

Total 2000  
 
Rating valuation matrix
Years 0-1 1-2 2-3 3-4
Rating
AAA 6.25% 6.75% 7.35% 8.00%
AA 6.75% 7.35% 8.05% 8.80%
A 7.75% 8.45% 9.15% 9.85%  
 
 
a. 14 years, 46.1% 
b. 2.3 years, 7.5% 
c. 2.3 years, 7.7% 
d. 4.4 years, 15.4%% 

4. An investment bank uses the Exponentially Weighted Moving Average (EWMA) technique with 
lambda of 0.9 to model the daily volatility of a security. The current estimate of the daily volatility is 
1.5%. The closing price of the security is USD 20 yesterday and USD 18 today. Using continuously‐
compounded returns, what is the updated estimate of the volatility? 
a. 5.44% 
b. 3.62% 
c. 2.96% 
d. 1.31% 
 
 
 
 

 
Copyright © 2009 Global Association of Risk Professionals         50    
All rights reserved.
2009 FRM Practice Exams 

5. Consider two stocks A and B.  Assume their annual returns are jointly normally distributed, the 
marginal distribution of each stock has mean 2% and standard deviation 10%, and the correlation is 
0.9.  What is the expected annual return of stock A if the annual return of stock B is 3%? 
 
a. 2.9% 
b. 2% 
c. 1.1% 
d. 4.7% 

6. In pricing a derivative using the Monte Carlo method, we need to simulate a reasonable number of 
paths for the price of the underlying asset.  Suppose we use a simple model for the return of the 
underlying asset: 

y(t) = drift*Δt + vol * √ Δt * e(t), and e(t) is distributed ~ N(0,1), 
 
where drift and vol are known parameters and Δt is the step size. 
 
The generation of each path requires a number of steps. Which of the following describes the 
correct procedure? 
 
a. Generate a random number from a normal distribution N(0,1), use the inverse normal function 
to get e(t), which will be fed into the model to get y(t). Repeat the same procedure until you get 
the full desired path. 
b. Generate a random number from a normal distribution N(0,1), use the cumulative normal 
function to get e(t), which will be fed into the model to get y(t). Repeat the same procedure 
until you get the full desired path. 
c. Generate a random number from a uniform distribution defined in [0,1], use the inverse 
cumulative normal function to get e(t), which will be fed into the model to get y(t). Repeat the 
same procedure until you get the full desired path. 
d. Generate a random number from a uniform distribution defined in [0,1], use the cumulative 
normal function to get e(t), which will be fed into the model to get y(t). Repeat the same 
procedure until you get the full desired path. 

7. Suppose that A and B are random variables, each follows a standard normal distribution, and the 
covariance between A and B is 0.35. What is the variance of (3A + 2B)? 
 
a. 5.10 
b. 14.47 
c. 9.20 
d. 17.20 

 
Copyright © 2009 Global Association of Risk Professionals         51    
All rights reserved.
2009 FRM Practice Exams 

8. You don’t have access to KMV’s data. Your boss wants you to tell him your estimate of the 
probability of default of a credit. To do so, you use the Merton Model because the credit you are 
considering has no systematic risk. In Merton’s Model, the distance to default (DD) and the 
expected default frequency (EDF) are 
 
a.  positively and linearly related 
b. negatively and linearly related 
c. positively and non‐linearly related 
d. negatively and non‐linearly related 

9. Suppose the rate on Company A’s one‐year zero‐coupon bond is 10.0% and the one‐year T‐bill rate 
is 8.0%.  Assume the T‐bill is riskless and the probability of default of Company A’s bond is 10%. 
What is the LGD of Company A’s bond? 
 
a. 18.18% 
b. 81.82% 
c. 20.01% 
d. 79.99% 
 
 
 
 
10. A bank is considering ways of significantly reducing or eliminating its credit exposure to defaults on a 
loan portfolio so that the bank’s shareholders do not absorb the losses arising from such defaults. 
Ignoring institutional issues (e.g., tax, accounting, capital requirements), three of the following 
programs have a similar impact on the credit risk of the bank. Which alternative fails to reduce 
credit risk? 
 
a. Sell the loan portfolio in its entirety to another bank. 
b. Borrow to finance an additional risk reserve to supplement existing loan‐loss reserves. 
c. Securitize the loan portfolio. 
d. Buy credit protection on the loan portfolio with credit default swaps. 
 
 
 
 
11. Consider a stock price S that follows a geometric Brownian motion dS = μ S dt + β S dz, with β
strictly positive and μ a fixed value.  Which of the following statements is true? 
 
a. If the drift μ is negative, the price one year from now will be below today’s price. 
b. The instantaneous rate of return on the stock follows a uniform distribution. 
c. The stock price S follows a lognormal distribution. 
d. This model imposes mean reversion. 

 
Copyright © 2009 Global Association of Risk Professionals         52    
All rights reserved.
2009 FRM Practice Exams 

12. The joint probability distribution of random variables X and Y is given by f(x,y) = kxy for x = 1, 2, 3, y = 
1, 2, 3, and k is a positive constant. What is the probability that X + Y will exceed 5? 
 
a. 1/9 
b. 1/4 
c. 1/36 
d. Cannot be determined 
 
 
 
 
13. Which of the following statements regarding Hypothesis Testing is incorrect? 
 
a. Hypothesis testing is used to make inferences about the parameters of a given population on 
the basis of statistics computed for a sample that is drawn from that population. 
b. Type II error refers to the failure to reject the null hypothesis when it is actually false. 
c. The p‐value decision rule is to reject the null hypothesis if the p‐value is greater than the 
significance level. 
d. All else being equal, the decrease in the chance of making a Type I error comes at the cost of 
increasing the probability of making a Type II error. 
 
 
 
 
14. If stock returns are independently identically normally distributed and the annual volatility is 30%, 
then the daily VaR at the 99% confidence level of a stock market portfolio is approximately: 
 
a. 2.41% 
b. 3.11% 
c. 4.40% 
d. 1.89% 
 
 
 
 
15. A single stock has a price of USD 10 and a current daily volatility of 2%. Using the delta‐normal 
method, the VaR at the 95% confidence level of a long at‐the‐money call on this stock over a 1‐day 
holding period is approximately: 
 
a. USD 0.23 
b. USD 1.645 
c. USD 0.33 
d. USD 0.16 
 
 
 
 

 
Copyright © 2009 Global Association of Risk Professionals         53    
All rights reserved.
2009 FRM Practice Exams 

16. A portfolio consists of two zero coupon bonds, each with a current value of USD 10. The first bond 
has a modified duration of 1 year and the second has a modified duration of 9 years. The yield curve 
is flat and all yields are 5%. Assume all moves of the yield curve are parallel shifts. Given that the 
daily volatility of the yield is 1%, which of the following is the best estimate of the portfolio daily VaR 
at the 95% confidence level? 
 
a. USD 2.33 
b. USD 1.65 
c. USD 0.82 
d. USD 1.16 
 
 
 
 
17. Consider the following three methods of estimating the P&L of a bullet bond: full repricing, duration 
(PV01), and duration plus convexity.  Ranking the estimated P&L impact of a large negative yield 
shock from the lowest P&L impact to the highest P&L impact, what is the ranking of the methods to 
estimate the P&L impact?  
 
a. duration plus convexity, duration, full repricing 
b. full repricing, duration plus convexity, duration 
c. duration, duration plus convexity, full repricing 
d. duration, full repricing, duration plus convexity 
 
 
 
18. Consider a position in a 5‐year receive‐fixed swap that makes annual payments on a USD 100 million 
notional.  The floating leg has just been reset.  The term structure is flat at 5%, the Macaulay 
duration of a 5‐year par bond is 4.5 years, and the annual volatility of yield changes is 100bp.  Your 
best estimate of the swap’s VaR with 95% confidence over the next month is 
 
a. USD 1.6 million 
b. USD 2.0 million 
c. USD 5.5 million 
d. USD 7.1 million 

19. If the gold lease rate is higher than the risk‐free rate, what is the market structure of the forward 
market for gold? 
 
a. Contango 
b. Backwardation 
c. Inversion 
d. Need more information to determine 

 
Copyright © 2009 Global Association of Risk Professionals         54    
All rights reserved.
2009 FRM Practice Exams 

20. The price of a 3‐year zero coupon government bond is 85.16.  The price of a similar 4‐year bond is 
79.81.  What is the one‐year implied forward rate from year 3 to year 4? 
 
a. 5.4% 
b. 5.5% 
c. 5.8% 
d. 6.7% 
 
 
 
 
21. A portfolio manager has a bond position worth USD 100 million.  The position has a modified 
duration of 8 years and a convexity of 150 years.  Assume that the term structure is flat.  By how 
much does the value of the position change if interest rates increase by 25 basis points? 
 
a. USD ‐1,953,125 
b. USD ‐1,906,250 
c. USD ‐2,046,875 
d. USD ‐2,187,500  
 
 
 
 
 
22. What is the annualized rate of return earned on a cash‐and‐carry trade entered into in March and 
closed out in June? 
 
a. 8.9% 
b. 9.8% 
c. 35.7% 
d. 39.1% 
 
 
 
 
23. An investor sells a June 2008 call of ABC Limited with a strike price of USD 45 for USD 3 and buys a 
June 2008 call of ABC Limited with a strike price of USD 40 for USD 5. What is the name of this 
strategy and the maximum profit and loss the investor could incur? 
 
a. Bear Spread, Maximum Loss USD 2, Maximum Profit USD 3 
b. Bull Spread, Maximum Loss Unlimited, Maximum Profit USD 3 
c. Bear Spread, Maximum Loss USD 2, Maximum Profit Unlimited 
d. Bull Spread, Maximum Loss USD 2, Maximum Profit USD 3 
 
 
 

 
Copyright © 2009 Global Association of Risk Professionals         55    
All rights reserved.
2009 FRM Practice Exams 

24. Which of the following problems are NOT inherent disadvantages of the historical simulation 
approach to estimating VaR? 
 
I.       It gives too little weight to more recent observations 
II. For long‐only portfolios, it is likely to understate VaR following a recent structural increase in 
volatilities 
III. It always ignores the fat tails present in the distribution of returns on many financial assets 
IV. Because of the delta approximation, it inadequately measures the risk of nonlinear 
instruments 
 
a. I and II only 
b. II only 
c. I, III and IV only 
d. III and IV only 
 
 
 
 
 
 
 
25. A bank holds USD 60 million worth of 10‐year 6.5% coupon bonds that are trading at a clean price of 
101.82. The bank is worried by the exposure due to these bonds but cannot unwind the position for 
fear of upsetting the client. Therefore, it purchases a total return swap (TRS) in which it receives 
annual Libor + 100 bps in return for the mark‐to‐market return on the bond.  For the first year, the 
Libor sets at 6.25% and by the end of the year the clean price of the bonds is at 99.35. The net 
receipt/payment for the bank in the total return swap will be: 
 
a. Receive USD 2.23 million 
b. Receive USD 1.93 million 
c. Pay USD 1.93 million 
d. Pay USD 2.23 million 

 
Copyright © 2009 Global Association of Risk Professionals         56    
All rights reserved.
2009 FRM Practice Exams 

26. Which of the following trade(s) contain basis risk? 
 
I.      Long 1,000 lots Nov 07 ICE Brent Oil contracts and short 1,000 lots Nov 07 NYMEX WTI Crude 
Oil contracts 
II. Long 1,000 lots Nov 07 ICE Brent Oil contracts and long 2,000 lots Nov 07 ICE Brent Oil at‐the‐
money put 
III. Long 1,000 lots Nov 07 ICE Brent Oil contracts and short 1,000 lots Dec 07 ICE Brent Oil 
contracts 
IV. Long 1,000 lots Nov 07 ICE Brent Oil contracts and short 1,000 lots Dec 07 NYMEX WTI Crude 
Oil contracts 
 
a. I & III 
b. II & IV 
c. IIII & IV 
d. I, III & IV 
 
 
 
 
 
27. According to put‐call parity, buying a put option on a stock is equivalent to: 
 
a. Buying a call option and buying the stock with funds borrowed at the risk‐free rate. 
b. Selling a call option and buying the stock with funds borrowed at the risk‐free rate. 
c. Buying a call option, selling the stock and investing the proceeds  at the risk‐free rate. 
d. Selling a call option, selling the stock and investing the proceeds at the risk‐free rate. 
 
 
 
 
 
28. A 3 month futures contract on an equity index is currently priced at USD 1000, the underlying index 
stocks are valued at USD 990 and pay dividends at a continuously‐compounded rate of 2 percent 
and the current continuously compounded risk‐free rate is 4 percent. The potential arbitrage profit 
per contract, given this set of data, is closest to 
 
a. USD 10.00 
b. USD 7.50 
c. USD 5.00 
d. USD 1.50 
 
 
 
 

 
Copyright © 2009 Global Association of Risk Professionals         57    
All rights reserved.
2009 FRM Practice Exams 

29. Research and model projections indicate that a specific event is likely to move the CHF against the 
USD. While the direction of the move is highly uncertain, it is highly likely that magnitude of the 
move will be significant. Based on this information, which of the following strategies would provide 
the largest economic benefit? 
 
a. Long a call option on USD/CHF and short a put option on USD/CHF with the same strike price 
and expiration date 
b.  Long a call option on USD/CHF and long a put option on USD/CHF with the same strike price and 
expiration date 
c.  Short a call option on USD/CHF and long a put option on USD/CHF with the same strike price 
and expiration date 
d. Short a call option on USD/CHF and short a put option on USD/CHF with the same strike price 
and expiration date 
 
 
 
 
30. Initially, the call option on Big Kahuna Inc. with 90‐days to maturity trades at USD 1.40. The option 
has a delta of 0.5739. A dealer sells 200 call option contracts and to delta‐hedge the position, the 
dealer purchases 11,478 shares of the stock at the current market price of USD 100 per share. The 
following day, the prices of both the stock and the call option increase. Consequently, delta 
increases to 0.7040. To maintain the delta hedge, the dealer should: 
 
a. Purchase 2,602 shares 
b. Sell 2,602 shares 
c. Purchase 1,493 shares 
d. Sell 1,493 shares 
 
 
 
 
31. Which of the following strategies creates a calendar spread? 
 
a. Sell a call option with a certain strike price and buy a longer maturity call option with the same 
strike price. 
b. Buy a call option with a certain strike price and buy a longer maturity call option with the same 
strike price. 
c. Sell a call option with a certain strike price and buy a shorter maturity call option with the same 
strike price. 
d. Buy a call option with a certain strike price and sell a longer maturity call option with the same 
strike price. 
 
 
 
 

 
Copyright © 2009 Global Association of Risk Professionals         58    
All rights reserved.
2009 FRM Practice Exams 

32. Which of the following underlying macro‐economic conditions would leave an emerging market 
most vulnerable to the contagion effects of a currency crisis?  
 
a. Large current account surplus, low foreign exchange reserves, non‐convertible currency 
b. Large current account deficit, low foreign exchange reserves, fully convertible currency 
c. Small current account deficit, high foreign exchange reserves, non‐convertible currency 
d. Large current account surplus, high foreign exchange reserves, fully convertible currency 
 
 
 
33. Consider an FRA (forward rate agreement) with the same maturity and compounding frequency as a 
Eurodollar futures contract. The FRA has a LIBOR underlying. Which of the following statements are 
true about the relationship between the forward rate and the futures rate? 
 
a. They should be exactly the same. 
b. The forward rate is normally higher than the futures rate. 
c. The forward rate is normally lower than the futures rate. 
d. They have no fixed relationship. 
 
 
 
34. Your bank is an active player in the commodity market. The view of the economist of the bank is 
that inflation is expected to rise moderately in the near term and market volatility is expected to 
remain low. The traders are advised to undertake deals on the metals exchange to align your book 
to conform with the expectations of the economist of the bank. As risk manager, you are asked to 
monitor the positions of the traders to make sure that they have the exposures to inflation and 
market volatility sought by the bank. Which trader has taken an appropriate position among the 
traders you are monitoring?  
 
a. Trader A bought a call and a put, both with 90‐days to expiration and with strike price equal to 
the existing spot level. 
b. Trader B bought a put option with a down‐and‐in knock in feature. 
c. Trader C bought a call option at the existing spot levels and sold a call at a higher strike price, 
both with 90‐days to expiration. 
d. Trader D sold a call and bought a put at the existing levels, both with 90‐days to expiration. 
 
 
 
 
35. Considering options generally (i.e., not only plain vanilla calls and puts), which of the following 
statements about vega is correct?  
 
a.  An option holder can never be vega negative. 
b.  A deep in the money up and out call option has a negative vega. 
c.  A deep out of the money up and out call option has a negative vega. 
d.  A deep out of the money digital option has a negative vega. 

 
Copyright © 2009 Global Association of Risk Professionals         59    
All rights reserved.
2009 FRM Practice Exams 

36. To hedge against future, unanticipated, and significant increases in borrowing rates, which of the 
following alternatives offers the greatest flexibility for the borrower? 
 
a. Fixed for floating swap 
b. Interest rate collar 
c. Interest rate floor 
d. Call swaption 
 
 
 
 
37. Assuming other things constant, bonds of equal maturity will still have different DV01 per USD 100 
face value.  Their DV01 per USD 100 face value will be in the following sequence of highest value to 
lowest value:   
 
a. Zero coupon bonds, par bonds, premium bonds 
b. Premium bonds, par bonds, zero coupon bonds 
c. Premium bonds, zero coupon bonds, par bonds 
d. Zero coupon bonds, premium bonds, par bonds 
 
 
 
 
38. The information ratio of the Sterole US Fund for 2006 against the S&P 500, its benchmark index, is 1.  
For the same time period, the fund’s Sharpe ratio is 2, the fund has a tracking error of 7% against 
the S&P 500, and the standard deviation of fund returns is 5%. The risk‐ free rate in the US is 4%. 
Calculate the return for the S&P 500 during the time period. 
 
a. 3.5% 
b. 7% 
c. 11% 
d. 14% 
 
 
 
39. A fund manager recently received a report on the performance of his portfolio over the last year. 
According to the report, the portfolio return is 9.3%, with a standard deviation of 13.5%, and a beta 
of 0.83. The risk‐free rate is 3.2%, the semi‐standard deviation σL(Rp) of the portfolio is 8.4%, and the 
tracking error of the portfolio to the benchmark index is 2.8%. What is the difference between the 
value of the fund’s Sortino ratio (computed relative to the risk‐free rate) and its Sharpe ratio? 
 
a. 0.274 
b. 1.727 
c. 0.653 
d. ‐0.378 

 
Copyright © 2009 Global Association of Risk Professionals         60    
All rights reserved.
2009 FRM Practice Exams 

40. Your firm has no prior derivatives trades with its counterparty Super Bank. Your boss wants you to 
evaluate some trades she is considering. In particular, she wants to know which of the following 
trades will increase your firm’s credit risk exposure to Super Bank: 
 
I.             buying a put option 
II. selling a put option 
III. buying a forward contract 
IV. selling a forward contract 
 
a. I. and II only 
b. II and IV only 
c. III and IV only 
d. I, III and IV only 
 
 
 
 
41. Consider the following one‐period transition matrix: 
 
Initial    Next Period State  
Period    A  B  Default
State  A  95% 5%  0% 
B  10% 80% 10% 
  Default 0%  0%  100% 
 
If a company is originally in State A, what is the probability that the company will have defaulted 
strictly before the fourth transition period from now? 
 
a. 0.875% 
b. 0.500% 
c. 1.375% 
d. 1.875% 
 
 
 
 
42. As an approximation, it is true that 
 
a. Default swap spread = Return of a risky bond + Return of a risk‐free bond 
b. Default swap spread = Return of a risky bond – Return of a risk‐free bond 
c. Default swap spread = Return of a risky bond x Return of a risk‐free bond 
d. Default swap spread = Return of a risky bond x (1 – Return of a risk‐free bond) 
 
   
 
 

 
Copyright © 2009 Global Association of Risk Professionals         61    
All rights reserved.
2009 FRM Practice Exams 

43. In a CDO, the SPV is typically 
 
a. AAA‐rated 
b. A‐rated 
c. BBB‐rated 
d. Not rated 
 
 
 
44. A trader whose risk you are monitoring tells you that he wants to benefit from a credit spread 
widening due to a recession.  Which of the following would be good trades for his strategy? 
 
a. Go long risky bonds and short risk‐free bonds at the beginning of the recession. 
b. Short risky bonds and go long risk‐free bonds at the beginning of the recession. 
c. Sell credit default swaps on bonds with a low credit quality and buy credit default swaps on 
bonds with a higher credit quality at the beginning of the recession. 
d. Sell credit default swaps on bonds with low credit quality and go long low credit quality bonds. 
 
 
 
45. Bank B has a EUR 100 million loan portfolio and has set aside a reserve to cover the first EUR 20 
million in default‐related losses. If the bank wants to acquire protection for the remaining EUR  80 
million in risk exposure, which of the following solutions would work and would expose the bank to 
the least amount of counterparty risk? 
 
a. Buy credit protection in a senior subordinated CDS that covers EUR  80 million in losses above 
the first EUR 20 million.   
b. Buy credit insurance for losses up to EUR 80 million in excess of EUR 20 million on the loan 
portfolio. 
c. Issue a credit‐linked note in which interest and principal may be withheld from investors to 
cover up to EUR 80 million in losses above the first EUR 20 million on the loan portfolio. 
d. All three of the above choices work and expose the bank to the same amount of counterparty 
risk. 
 
 
 
 
46. Mr. Rosenqvist, Asset Manager at BCD Bank, holds a portfolio of SEK 200 million. The portfolio 
consists of BBB‐rated bonds. Assume that the one‐year probability of default is 4%, the recovery 
rate is 60%, and defaults are uncorrelated over years. What is the 2‐year cumulative expected credit 
loss on Mr. Rosenqvist’s portfolio? 
 
a. SEK 6.35 million 
b. SEK 6.40 million 
c. SEK 9.48 million 
d. SEK 9.60 million 

 
Copyright © 2009 Global Association of Risk Professionals         62    
All rights reserved.
2009 FRM Practice Exams 

47. Using the Merton model, the value of the debt increases if all other parameters are fixed and 
 
I.            The value of the firm decreases 
II. The riskless interest rate decreases 
III. Time to maturity increases 
IV. The volatility of the firm value decreases 
 
a. I and II only 
b. I and IV only 
c. II and III only 
d. II and IV only 
 
 
 
 
48. A firm is going to buy 10,000 barrels of West Texas Crude Oil.  It plans to hedge the purchase using 
the Brent Crude futures contract.  The correlation between the spot and futures prices is 0.72.  The 
volatility of the spot price is 0.35 per year.  The volatility of the Brent Crude futures price is 0.27 per 
year.  What is the hedge ratio for the firm? 
 
a. 0.5554 
b. 0.9333 
c. 1.2099 
d. 0.8198 
 
 
 
 
49. It is June 2nd and a fund manager with USD 10 million invested in government bonds is concerned 
that interest rates will be highly volatile over the next three months.  The manager decides to use 
the September Treasury bond futures contract to hedge the value of the portfolio.  The current 
futures price is 95.0625.  Each contract is for the delivery of USD 100,000 face value of bonds.  The 
duration of the manager’s bond portfolio in three months will be 7.8 years.  The cheapest to deliver 
bond in the Treasury bond futures contract is expected to have a duration of 8.4 years at maturity of 
the contract.  At the maturity of the Treasury bond futures contract, the duration of the underlying 
benchmark Treasury bond is 9 years. What position should the fund manager undertake to mitigate 
his interest rate risk exposure? 
 
a. Short94 contracts 
b. Short98 contracts 
c. Short105 contracts 
d. Short113 contracts 
 
 
 
 

 
Copyright © 2009 Global Association of Risk Professionals         63    
All rights reserved.
2009 FRM Practice Exams 

50. A bond trader has bought a position in Treasury Bonds with a 4% annual coupon rate on February 
15, 2015. The DV01 of the position is USD 80,000. The trader decides to hedge his interest rate risk 
with the 4.5% coupon rate Treasury Bonds maturing on May 15, 2017 which has a DV01 of .076 per 
USD 100 face value. To implement this hedge, approximately what face amount of the 4.5% 
Treasury bonds maturing on May 15, 2017 should the trader sell? 
 
a. USD 80,000 
b. USD 10,500,000 
c. USD 80,000,000 
d. USD 105,000,000 
 
 
 
 
 
 
 
 
END OF 2009 FRM FULL EXAM PRACTICE EXAM I 
 
 
 

 
Copyright © 2009 Global Association of Risk Professionals         64    
All rights reserved.
2009 FRM Practice Exams 

2009 FRM Full Exam Practice Exam I  
Answer Key 
 
 
1. a. b. c. d. 26. a. b. c. d.
2. a. b. c. d. 27. a. b. c. d.
3. a. b. c. d. 28. a. b. c. d.
4. a. b. c. d. 29. a. b. c. d.
5. a. b. c. d. 30. a. b. c. d.
6. a. b. c. d. 31. a. b. c. d.
7. a. b. c. d. 32. a. b. c. d.
8. a. b. c. d. 33. a. b. c. d.
9. a. b. c. d. 34. a. b. c. d.
10. a. b. c. d. 35. a. b. c. d.
11. a. b. c. d. 36. a. b. c. d.
12. a. b. c. d. 37. a. b. c. d.
13. a. b. c. d. 38. a. b. c. d.
14. a. b. c. d. 39. a. b. c. d.
15. a. b. c. d. 40. a. b. c. d.
16. a. b. c. d. 41. a. b. c. d.
17. a. b. c. d. 42. a. b. c. d.
18. a. b. c. d. 43. a. b. c. d.
19. a. b. c. d. 44. a. b. c. d.
20. a. b. c. d. 45. a. b. c. d.
21. a. b. c. d. 46. a. b. c. d.
22. a. b. c. d. 47. a. b. c. d.
23. a. b. c. d. 48. a. b. c. d.
24. a. b. c. d. 49. a. b. c. d.
25. a. b. c. d. 50. a. b. c. d.

 
Copyright © 2009 Global Association of Risk Professionals         65    
All rights reserved.
2009 FRM Practice Exams 

 
 
 
 
 
 
 
 
 
 
 
 
 
 
 
 
 
This page is intentionally left blank.  
 
 
 
 
 
 
 
 
 
 
 
 
 
 
 
 
 
 
 

 
Copyright © 2009 Global Association of Risk Professionals         66    
All rights reserved.
2009 FRM Practice Exams 

2009 FRM Full Exam Practice Exam I  
Answers & Explanations 
 
1. Given the information provided in the table below, what is the portfolio VaR, at the 99% confidence 
level, of the following 100 million CHF equally weighted investment portfolio?  
 
Expected 
Asset  Volatility Correlation 
Return 
      Stocks  Bonds 
Stocks  24.00%  18%  1   
Bonds  15.00%  6%  0.1  1 
 
a. 27.96 million CHF 
b. 22.77 million CHF 
c. 20.97 million CHF 
d. 13.98 million CHF 
 
CORRECT: B 
 
The variance of the equally weighted portfolio is 0.5^2 * 0.18^2 + 0.5^2 * 0.06^2 + 2 * 0.5 * 0.5 * 0.1 
* 0.18 * 0.06 = 0.081 + 0.0009 + 0.0005 = 0.00954. The volatility is then 9.77%. The portfolio VaR or 
the risk budget is 2.33 * 9.77% * 100 million CHF = 22.77 million CHF. 
 
Reference: Allen et al. Chapters 2,3. 
 
 
 
 
2. You are asked by your boss to estimate the exposure of a hedge fund to the S&P 500. Though the 
fund claims to mark to market weekly, it does not do so and marks to market once a month. The 
fund also does not tell investors that it simply holds an ETF which is indexed to the S&P500. Because 
of the claims of the hedge fund, you decide to estimate the market exposure by regressing weekly 
returns of the fund on the weekly return of the S&P500. Which of the following properties correctly 
describes a property of your regression estimates?  
 
a. The beta of your regression will be one because the fund holds the S&P 500. 
b. The beta of your regression will be zero because the fund returns are not synchronous with the 
S&P 500 returns. 
c. The intercept of your regression will be positive, showing that the fund has a positive alpha 
when estimated using an OLS regression. 
d. The beta will be misestimated because hedge fund exposures are non‐linear. 
 

 
Copyright © 2009 Global Association of Risk Professionals         67    
All rights reserved.
2009 FRM Practice Exams 

CORRECT: C 
 The alpha is spurious and results from the fact that returns are non‐synchronous. d. is incorrect 
because the true exposure is linear. The beta is greater than zero and less than one because of non‐
synchroneity. 

Reference: Amenc and Le Sourd, Portfolio Theory and Performance Analysis. Chapter 4 
 
 
 
 
3. The following table shows the composition of the GARP Bond Fund. What are the portfolio duration 
and portfolio yield of the fund? 
 
GARP Bond Fund
Rating Amount Duration
Mn USD in years
AAA
Company A 600 1.5
Company B 300 4
Company C 200 2.5
AA
Company D 400 4
Company E 350 0.5
A
Company F 150 1.5

Total 2000  
 
Rating valuation matrix
Years 0-1 1-2 2-3 3-4
Rating
AAA 6.25% 6.75% 7.35% 8.00%
AA 6.75% 7.35% 8.05% 8.80%
A 7.75% 8.45% 9.15% 9.85%  
 
 
a. 14 years, 46.1% 
b. 2.3 years, 7.5% 
c. 2.3 years, 7.7% 
d. 4.4 years, 15.4%% 
 
The calculation of portfolio duration and portfolio yield is based on the proportional weightage of 
respective company to its duration and yield. The portfolio duration and portfolio yield after mapping 
the yield from rating matrix is as follows; 
 
CORRECT: B 

 
Copyright © 2009 Global Association of Risk Professionals         68    
All rights reserved.
2009 FRM Practice Exams 

 This answer reflects the proportion of amount taken as weights to calculate the portfolio duration 
and portfolio yield. 
 
GARP Bond Fund
Amount Proportion % Duration Yield
AAA
Co A 600 30% 1.5 6.75%
Co B 300 15% 4 8.00%
Co C 200 10% 2.5 7.35%
AA
Co D 400 20% 4 8.80%
Co E 350 18% 0.5 6.75%
A
Co F 150 8% 1.5 8.45%

Total 2000 100% 2.30 7.54%


 
 
INCORRECT: A – If the candidate does a simple addition of the duration and the mapped yield, he 
would get this answer. 
INCORRECT: C – Though the portfolio duration is correct, but it is arrived by taking simple average of 
duration. However, if the candidate would take simple average of mapped yield instead of 
proportion his answer would be 7.7% and not 7.5% 
INCORRECT: D – If the candidate averages based on rating classes (3 – AAA/AA/A) instead of the 
companies, he would get this answer. 
 
Reference: Tuckman; Chapter 6. 
 
 
 
 
4. An investment bank uses the Exponentially Weighted Moving Average (EWMA) technique with 
lambda of 0.9 to model the daily volatility of a security. The current estimate of the daily volatility is 
1.5%. The closing price of the security is USD 20 yesterday and USD 18 today. Using continuously‐
compounded returns, what is the updated estimate of the volatility? 
 
a. 5.44% 
b. 3.62% 
c. 2.96% 
d. 1.31% 
 
CORRECT: B 
 
The current return of the security is = ln (18/20) = ‐10.536%. Using an EWMA model, the updated 
volatility is given as: V(t)   = {lambda* ((V[t‐1]^2) +(1 – lambda)*(current return^2)} ^ 0.5 
= {0.9 * ((0.015^2) + (1 ‐ 0.9) * ( ‐0.10536^2  )}  ^ 0.5 

 
Copyright © 2009 Global Association of Risk Professionals         69    
All rights reserved.
2009 FRM Practice Exams 

= 3.62% 
 
INCORRECT: A – Forgets to square the volatility terms 
INCORRECT: C – Forgets to square the volatility terms and to take the square root of the resulting 
variance, then miscalculates conversion to percentage. 
INCORRECT: D – Forgets to take the square root of the variance, then miscalculates conversion to 
percentage. 
 
Reference : Hull, Chapter 21. 
 
 
 
5. Consider two stocks A and B.  Assume their annual returns are jointly normally distributed, the 
marginal distribution of each stock has mean 2% and standard deviation 10%, and the correlation is 
0.9.  What is the expected annual return of stock A if the annual return of stock B is 3%? 
 
a. 2.9% 
b. 2% 
c. 1.1% 
d. 4.7% 
 
CORRECT: A 
 
E[ra | rb = x] = μa + (ρabσaσb/σ a)(x – μb) = 0.02 + 0.9 * (0.03 – 0.02) = 0.029 
 
Reference: Damodar Gujarati, chapter 2. 
 
 
 
 
6. In pricing a derivative using the Monte Carlo method, we need to simulate a reasonable number of 
paths for the price of the underlying asset.  Suppose we use a simple model for the return of the 
underlying asset: 
 
y(t) = drift*Δt + vol * √ Δt * e(t), and e(t) is distributed ~ N(0,1), 
 
where drift and vol are known parameters and Δt is the step size. 
 
The generation of each path requires a number of steps. Which of the following describes the 
correct procedure? 
 
a. Generate a random number from a normal distribution N(0,1), use the inverse normal function 
to get e(t), which will be fed into the model to get y(t). Repeat the same procedure until you get 
the full desired path. 

 
Copyright © 2009 Global Association of Risk Professionals         70    
All rights reserved.
2009 FRM Practice Exams 

b. Generate a random number from a normal distribution N(0,1), use the cumulative normal 
function to get e(t), which will be fed into the model to get y(t). Repeat the same procedure 
until you get the full desired path. 
c. Generate a random number from a uniform distribution defined in [0,1], use the inverse 
cumulative normal function to get e(t), which will be fed into the model to get y(t). Repeat the 
same procedure until you get the full desired path. 
d. Generate a random number from a uniform distribution defined in [0,1], use the cumulative 
normal function to get e(t), which will be fed into the model to get y(t). Repeat the same 
procedure until you get the full desired path. 
 
CORRECT: C 
 
This question wants to test if the candidate knows the basic steps to generate a very simple path: 
answering this question means that the candidate would be able to build a simple spreadsheet 
showing the Monte Carlo logic. The correct procedure is the one described in c); while a), b) and d) 
are nonsensical calculations. 
 
Reference: Philippe Jorion, Value at Risk,The New Benchmark for Managing Financial Risk, 3rd edition 
(New York: McGraw‐Hill, 2007), Chapter 12. 
 
 
 
7. Suppose that A and B are random variables, each follows a standard normal distribution, and the 
covariance between A and B is 0.35. What is the variance of (3A + 2B)? 
 
a. 15.10 
b. 14.47 
c. 9.20 
d. 17.20 
 
CORRECT: D 
 
 Since each variable is standardized, its variance is 1. Therefore V(3A+2B) = 9 V(A) + 4 V(B) + 2 x 3 x2 
x Cov(A,B) = 9+4+4.2 = 17.2 
 
INCORRECT: A –    9 + 4 + 6 * 0.35 = 15.1 
INCORRECT: B –    9+ 4 + 12 * 0.35^2 =  
INCORRECT: C –    3 + 2 + 12 * 0.35 = 9.2 
 
Reference: Damodar Gujarati 

 
Copyright © 2009 Global Association of Risk Professionals         71    
All rights reserved.
2009 FRM Practice Exams 

8. You don’t have access to KMV’s data. Your boss wants you to tell him your estimate of the 
probability of default of a credit. To do so, you use the Merton Model because the credit you are 
considering has no systematic risk. In Merton’s Model, the distance to default (DD) and the 
expected default frequency (EDF) are 
 
a. positively and linearly related 
b. negatively and linearly related 
c. positively and non‐linearly related 
d. negatively and non‐linearly related   
 
CORRECT: D 
 
The risk neutral probability of default, EDF, in the Merton Model is 1‐ N(d2). The higher the distance 
V 1
ln( − rT
) − σ 2V T
De 2
to default, DD DD ≡ d2 =  σV T , the lower the risk neutral probability of default is. On 
the contrary, the lower DD, the higher EDF is. The relationship is non‐linear. When the DD is low, EDF 
is high. If DD is imminent, EDF is high as well.  Similarly, if DD is high, EDF is small and not imminent 
 
Reference: De Servigny and Renault, Measuring and Managing Credit Risk, Chapter 3. 
 
 
 
 
9. Suppose the rate on Company A’s one‐year zero‐coupon bond is 10.0% and the one‐year T‐bill rate 
is 8.0%.  Assume the T‐bill is riskless and the probability of default of Company A’s bond is 10%. 
What is the LGD of Company A’s bond? 
 
a. 18.18% 
b. 81.82% 
c. 20.01%  
d. 79.99% 
 
CORRECT: A 
 
(1+10%)*(1‐PD)+(1+10%)*PD*(1‐LGD)=1+8% 
1.1 x 0.9 + 1.1 x 0.10 x (1 – LGD) = 1.08 
0.99 + 0.11 x (1 ‐ LGD) = 1.08 
0.11 x (1 ‐ LGD) = 1.08 – 0.99 
 (1 ‐ LGD) = (1.08 – 0.99) / 0.11 
LGD = 1‐ (1.08 – 0.99) / 0.11 = 18.18% 
 
or 

 
Copyright © 2009 Global Association of Risk Professionals         72    
All rights reserved.
2009 FRM Practice Exams 

 
LGD = 1 – ((1+rf) – (1+r) x (1 – PD))/((1+r) x PD) 
 
Reference: De Servigny and Renault, Measuring and Managing Credit Risk, Chapter 3, 4. 
 
 
 
 
10. A bank is considering ways of significantly reducing or eliminating its credit exposure to defaults on a 
loan portfolio so that the bank’s shareholders do not absorb the losses arising from such defaults. 
Ignoring institutional issues (e.g., tax, accounting, capital requirements), three of the following 
programs have a similar impact on the credit risk of the bank. Which alternative fails to reduce 
credit risk? 
 
a. Sell the loan portfolio in its entirety to another bank. 
b. Borrow to finance an additional risk reserve to supplement existing loan‐loss reserves. 
c. Securitize the loan portfolio. 
d. Buy credit protection on the loan portfolio with credit default swaps. 
 
CORRECT: B  
 
All three of the other choices are economically equivalent. Selling loans to an external party 
eliminates all credit risk for the institution.  Similarly, securitizing the loan portfolio removes the 
loans from the bank’s books and eliminates the credit risk for the institution.  Buying credit 
protection using credit default swaps will offer protection against credit risk. This alternative implies 
counterparty risk.  Borrowing does not work in the long run because shareholders still at some point 
have to take the hit for default‐related losses.  Additionally, the increased borrowing to finance the 
loan loss reserves will increase the risk for the shareholders.  
 
Reference: Culp, Chapter 16 
 
 
 
 
11. Consider a stock price S that follows a geometric Brownian motion dS = μ S dt + β S dz, with β strictly 
positive and μ a fixed value.  Which of the following statements is true? 
 
a. If the drift μ is negative, the price one year from now will be below today’s price. 
b. The instantaneous rate of return on the stock follows a uniform distribution. 
c. The stock price S follows a lognormal distribution. 
d. This model imposes mean reversion. 
 
CORRECT: C 
 

 
Copyright © 2009 Global Association of Risk Professionals         73    
All rights reserved.
2009 FRM Practice Exams 

INCORRECT: A – The expected price is less than today’s price, but not the price in all the states of 
world. 
INCORRECT: B – The instantaneous rate of return on the stock follows normal distribution.  
INCORRECT: D – This model does not impose mean reversion. 
 
rd 
Reference:  Philippe Jorion, Value at Risk: The New Benchmark for Managing Financial Risk, 3 ed. 
(New York: McGraw‐Hill, 2007). Chapter 12 
 
 
 
 
12. The joint probability distribution of random variables X and Y is given by f(x,y) = kxy for x = 1, 2, 3, y = 
1, 2, 3, and k is a positive constant. What is the probability that X + Y will exceed 5? 

 
a. 1/9 
b. 1/4 
c. 1/36 
d. Cannot be determined 
 
CORRECT: B 
3 3
Note that  ∑∑ f ( x, y) = 1 . 
x =1 y =1

Substituting the various values of x and y, we get f(1,1)=k, f(1,2)=2k, f(1,3)=3k, f(2,1)=2k, f(2,2)=4k, 
f(2,3)=6k, f(3,1)=3k, f(3,2)=6k, and f(3,3)=9k. Therefore, 
   k1 + 2k + 3k + 2k + 4k + 6k + 3k + 6k + 9k = 1 
so that, 36k = 1 and k=1/36. 
P(X+Y>5) = f(3,3) = 1/36 x 3 x 3 = 1/4 
 
Reference: Damodar Gujarati 
 
 
 
 
13. Which of the following statements regarding Hypothesis Testing is incorrect? 

 
a. Hypothesis testing is used to make inferences about the parameters of a given population on 
the basis of statistics computed for a sample that is drawn from that population. 
b. Type II error refers to the failure to reject the null hypothesis when it is actually false. 
c. The p‐value decision rule is to reject the null hypothesis if the p‐value is greater than the 
significance level. 
d. All else being equal, the decrease in the chance of making a Type I error comes at the cost of 
increasing the probability of making a Type II error. 

 
Copyright © 2009 Global Association of Risk Professionals         74    
All rights reserved.
2009 FRM Practice Exams 

 
CORRECT: C 
 
The true statement is to reject Ho if the p‐value is smaller than the significance level. 
 
INCORRECT: A – regarding the primary use of Hypothesis Testing. 
INCORRECT: B – regarding the definition of type II error. 
INCORRECT: D – type I error and type II error are in tradeoff. 
 
Reference: Damodar Gujarati 
 
 
 
 
14. If stock returns are independently identically normally distributed and the annual volatility is 30%, 
then the daily VaR at the 99% confidence level of a stock market portfolio is approximately: 
 
a. 2.41% 
b. 3.11% 
c. 4.40% 
d. 1.89% 
 
CORRECT: C 
 
The 1‐day volatility is s * (1/252)^0.5 = 0.3 * 0.629941 = 0.018898. The VaR at the 99% confidence 
level is then equal to 2.32635 * 0.018898 = 4.40% 
 
INCORRECT: A – One gets A if one uses 1.645 instead of 2.326,  
INCORRECT: B – One gets B if one uses the monthly volatility instead of the daily one 
INCORRECT: D – One gets D is the daily volatility. 
 
Reference: Allen, Boudoukh and Saunders, 2004, chapter 1, p 6‐8 
 
 
 
15. A single stock has a price of USD 10 and a current daily volatility of 2%. Using the delta‐normal 
method, the VaR at the 95% confidence level of a long at‐the‐money call on this stock over a 1‐day 
holding period is approximately: 
 
a. USD 0.23 
b. USD 1.645 
c. USD 0.33 
d. USD 0.16 
 
CORRECT: D 

 
Copyright © 2009 Global Association of Risk Professionals         75    
All rights reserved.
2009 FRM Practice Exams 

This question requires candidates to know the formula for the delta‐normal VaR approximation, and 
also to know that the delta of an at‐the‐money call is 0.5. 
 
VaR =| Δ | ×1.645 × σ × S = 0.5 × 1.645 × 0.02 × 10 = 0.1645 . 
 
INCORRECT: A – We get A by using 2.326 instead of 1.645 
INCORRECT: B – We get B if we use 2 instead of 2% for the volatility 
INCORRECT: C – We get C if we use a delta of 1 
 
Reference: Allen et al, Chapter 3 
 
 
 
 
16. A portfolio consists of two zero coupon bonds, each with a current value of USD 10. The first bond 
has a modified duration of 1 year and the second has a modified duration of 9 years. The yield curve 
is flat and all yields are 5%. Assume all moves of the yield curve are parallel shifts. Given that the 
daily volatility of the yield is 1%, which of the following is the best estimate of the portfolio daily VaR 
at the 95% confidence level? 
 
a. USD 2.33. 
b. USD 1.65. 
c. USD 0.82. 
d. USD 1.16 
 
CORRECT: B 
 
This question assesses candidates’ abilities to apply the duration VaR formula to two bonds 
simultaneously and to recall that the duration of a zero coupon bond is equal to the bond maturity.  
Using an obvious extension of Jorion’s equation 9.5 
 
VaR = D1 × V1 × 1.645 × σ + D2 × V2 × 1.645 × σ = ( D1 × V1 + D2 × V2 ) × 1.645 × σ =  
( D1 + D2 ) × 10 × 1.645 × σ = 10 × 10 × 1.645 × 0.01 = 1.645  
 
INCORRECT: A – Is the 99% confidence level VaR 
INCORRECT: C – Arises if the candidate mistakenly divides the correct answer by the number of bonds  
INCORRECT: D – Makes both mistakes 
 
Reference: Tuckman 
 

 
Copyright © 2009 Global Association of Risk Professionals         76    
All rights reserved.
2009 FRM Practice Exams 

17. Consider the following three methods of estimating the P&L of a bullet bond: full repricing, duration 
(PV01), and duration plus convexity.  Ranking the estimated P&L impact of a large negative yield 
shock from the lowest P&L impact to the highest P&L impact, what is the ranking of the methods to 
estimate the P&L impact?  
 
a. duration plus convexity, duration, full repricing 
b. full repricing, duration plus convexity, duration 
c. duration, duration plus convexity, full repricing 
d. duration, full repricing, duration plus convexity  
 
CORRECT: C 
 
The price / yield line with yield on the x axis and price on the y axis is convex to the origin.  The 
duration at any yield level is the tangent to that curve.  Therefore, except at the exact point of 
tangency, duration will always underestimate the price change. 
 
INCORRECT: A – Duration will always underestimate price change for negative yield shocks 
INCORRECT: B – Full repricing will never generate a smaller positive price change than duration 
because duration represents the point of tangency 
INCORRECT: D – Full repricing will generate a higher price for a large negative yield change than wil 
duration plus convexity 
 
Reference: Allen, Boudoukh, Saunders, Chapter 3 
 
 
 
18. Consider a position in a 5‐year receive‐fixed swap that makes annual payments on a USD 100 million 
notional.  The floating leg has just been reset.  The term structure is flat at 5%, the Macaulay 
duration of a 5‐year par bond is 4.5 years, and the annual volatility of yield changes is 100bp.  Your 
best estimate of the swap’s VaR with 95% confidence over the next month is 

 
a. USD1.6 million 
b. USD 2.0 million 
c. USD 5.5 million 
d. USD 7.1 million 
 
CORRECT: A 
 
 Because the floating‐rate leg has just been reset, its duration is 1.  Net duration is 4.5‐1=3.5 year, or 
modified duration of 3.5/1.05=3.33.  The 95% VaR of monthly changes in yields is 1.65*1%/√12 = 
0.48%.  Multiplying, this gives USD 100*0.48%*3.33=USD 1.588 
 

 
Copyright © 2009 Global Association of Risk Professionals         77    
All rights reserved.
2009 FRM Practice Exams 

INCORRECT: B – This uses a net duration of 4.5 years and ignores the duration of the floating‐rate 
leg. 
INCORRECT: C – This is the annual VaR, but should be translated to a monthly horizon. 
INCORRECT: D – This is the annual VaR computed by ignoring the duration of the floating‐rate leg. 
 
Reference: Hull, Chapter, Chapter 7 
 
 
 
19. If the gold lease rate is higher than the risk‐free rate, what is the market structure of the forward 
market for gold? 
 
a. Contango 
b. Backwardation 
c. Inversion 
d. Need more information to determine  
  
 CORRECT: B 
 
A lease rate higher than the risk fee rate will force a negatively sloped forward curve, i.e. 
backwardation 
  
INCORRECT: A – The forward price = spot*exp( risk free rate ‐ lease rate).  If the lease rate is higher 
than the risk free rate, forwards will be lower than spot, implying contango 
INCORRECT: C – The term inversion is used to describe yield curves, not commodity forwards 
INCORRECT: D – There is enough information in the question to provide an answer 
 
Reference: MacDonald, Chapter 6 
 
 
 
20. The price of a 3‐year zero coupon government bond is 85.16.  The price of a similar 4‐year bond is 
79.81.  What is the one‐year implied forward rate from year 3 to year 4? 
 
a. 5.4% 
b. 5.5% 
c. 5.8% 
d. 6.7% 
 
CORRECT: D 
 

 
Copyright © 2009 Global Association of Risk Professionals         78    
All rights reserved.
2009 FRM Practice Exams 

Price of three bond 85.16


1 + Forward rate = = = 1.067034
Price of four year bond 79.81
 
Forward rate = 0.067034 or 6.7%
 
INCORRECT: A – Is a combination of B and C 
INCORRECT: B – Is the return of the 3‐year bond 
INCORRECT: C – Is the return of the 4‐year bond 
  
Reference: Hull, Tuckman 
 
 
21. A portfolio manager has a bond position worth USD 100 million.  The position has a modified 
duration of 8 years and a convexity of 150 years.  Assume that the term structure is flat.  By how 
much does the value of the position change if interest rates increase by 25 basis points? 
 
a. USD ‐1,953,125 
b. USD ‐1,906,250 
c. USD ‐2,046,875 
d. USD ‐2,187,500  
 
CORRECT:  A 
 
ΔV = - D mod × Δy × V + 0.5 × Convexity × Δy 2 × V
ΔV = - 8 × 0.0025 × 100M + 0.5 × 150 × (0.0025) 2 × 100M
 
ΔV = - 2M + 46,875
ΔV = - 1,953,125
 
INCORRECT: B – Omits 0.5 from the second term 
INCORRECT: C – Subtracts the second term 
INCORRECT: D – Makes both mistakes 
 
Reference: Tuckman 
 
 
22. What is the annualized rate of return earned on a cash‐and‐carry trade entered into in March and 
closed out in June? 
 
a. 8.9% 
b. 9.8% 
c. 35.7% 
d. 39.1% 

 
Copyright © 2009 Global Association of Risk Professionals         79    
All rights reserved.
2009 FRM Practice Exams 

 
CORRECT: C 
 
By formula F0,T = S0erT + C, where F0,T = June forward price, S0 = March forward price, r = risk free 
interest rate, T = length of cash‐and‐carry, C = storage cost 
Solving 5.90 = 5.35er*3/12 + 0.05 
Solution is r = 35.7% 
 
INCORRECT: A – 8.9 = LN((5.9‐0.05)/5.35) (forgets to annualize the return) 
INCORRECT: B – 9.8 = LN((5.9)/5.35) (forgets to include the storage cost and to annualize the return) 
INCORRECT: D – 39.1 = (12/3)LN((5.9)/5.35) – 0.05 (forgets to include the storage cost) 
 
Reference: Robert L McDonald, Derivatives Markets, Chapter 6 
 
 
 
 
23. An investor sells a June 2008 call of ABC Limited with a strike price of USD 45 for USD 3 and buys a 
June 2008 call of ABC Limited with a strike price of USD 40 for USD 5. What is the name of this 
strategy and the maximum profit and loss the investor could incur? 
 
a. Bear Spread, Maximum Loss USD 2, Maximum Profit USD 3 
b. Bull Spread, Maximum Loss Unlimited, Maximum Profit USD 3 
c. Bear Spread, Maximum Loss USD 2, Maximum Profit Unlimited 
d. Bull Spread, Maximum Loss USD 2, Maximum Profit USD 3 
 
CORRECT: D 
 
Buying a call option at lower stock price and selling call option at higher strike price is called as Bull 
Spread. Bear Spread is buying the call option at higher price and selling the call at lower strike price. 
The Cost of strategy will be USD 3‐USD 5 = ‐USD 2 
The Payoff, when Stock price ST ≤ USD 40 will be ‐USD 2 (the cost of strategy) as none of the option 
will be exercised. 
The Payoff, when stock price ST ≥ 45, (as both options will be exercise) will be USD 5, 
Since the cost of strategy is ‐USD 3, hence profit will be USD 5‐USD 2 = USD 3 
When Stock price is USD 40< ST > USD 45, Only the call option bought by the investor would be 
exercised hence the pay off will be ST – 40, since the cost of strategy is ‐USD 3, The Net profit will be 
ST – 43, which would always be lower than USD 3. 
 
Reference: Hull, Chapter 10‐ Trading Strategies Involving Options.  
 

 
Copyright © 2009 Global Association of Risk Professionals         80    
All rights reserved.
2009 FRM Practice Exams 

24. Which of the following problems are NOT inherent disadvantages of the historical simulation 
approach to estimating VaR? 
 
I.      It gives too little weight to more recent observations 
II. For long‐only portfolios, it is likely to understate VaR following a recent structural increase in 
volatilities 
III. It always ignores the fat tails present in the distribution of returns on many financial assets 
IV. Because of the delta approximation, it inadequately measures the risk of nonlinear 
instruments 
 
a. I and II only 
b. II only 
c. I, III and IV only 
d. III and IV only 
 
CORRECT: C 
 
The disadvantage with the Historical Simulation Model is that it may not recognize the changes in 
volatility and correlation following recent structural changes. The model can be adjusted so that it 
gives more weight to recent observations. The other options, i.e. III & IV, are disadvantages of Monte 
Carlo method and Delta‐normal method.  
 
Reference: Allen et al. Chapters 2,3. 
 
 
 
 
25. A bank holds USD 60 million worth of 10‐year 6.5% coupon bonds that are trading at a clean price of 
101.82. The bank is worried by the exposure due to these bonds but cannot unwind the position for 
fear of upsetting the client. Therefore, it purchases a total return swap (TRS) in which it receives 
annual Libor + 100 bps in return for the mark‐to‐market return on the bond.  For the first year, the 
Libor sets at 6.25% and by the end of the year the clean price of the bonds is at 99.35. The net 
receipt/payment for the bank in the total return swap will be: 

 
a. Receive USD 2.23 million. 
b. Receive USD 1.93 million. 
c. Pay USD 1.93 million. 
d. Pay USD 2.23 million. 
 
CORRECT: B 
 

 
Copyright © 2009 Global Association of Risk Professionals         81    
All rights reserved.
2009 FRM Practice Exams 

it’s the result of this calculation: the notional amount is 60 million USD . Therefore the bank will 
receive the interest payment linked to the LIBOR rate: 60 million USD  * (6,25%+100 bp) = 4. 35 
million USD . 
The bank will pay the fixed coupon plus the change in the value of the bond: 60 million USD  * 6.5% + 
60 million *(99.35%‐101.82%) = 2.418 million USD . 
Hence the total net amount the bank will receive is: 4.35 million USD  ‐ 2.418 million USD  = 1.932 
million USD . 
 
Reference: Hull Chapter 7 ‐ Swaps 
 

26. Which of the following trade(s) contain basis risk? 
 
I.            Long 1,000 lots Nov 07 ICE Brent Oil contracts and short 1,000 lots Nov 07 NYMEX WTI 
Crude Oil contracts 
II.  Long 1,000 lots Nov 07 ICE Brent Oil contracts and long 2,000 lots Nov 07 ICE Brent Oil at‐
the‐money put 
III. Long 1,000 lots Nov 07 ICE Brent Oil contracts and short 1,000 lots Dec 07 ICE Brent Oil 
contracts 
IV. Long 1,000 lots Nov 07 ICE Brent Oil contracts and short 1,000 lots Dec 07 NYMEX WTI 
Crude Oil contracts 
 
a. I & III 
b. II & IV 
c. IIII & IV 
d. I, III & IV 
 
CORRECT: D 
 
Basis Risk is spread risk, which arise from trading the spread (long and short 2 positively correlated 
assets or same asset with different expiration) 
 
I  is spread trade in highly correlated asset with same expiration month 
II faces with gamma and vega risk 
III is spread trade in trading the flattening of the forward curve 
IV is spread trade in trading 2 assets with different expiration date 
 
Reference: Robert L. McDonald, Derivatives Markets (Boston: Addison‐Wesley, 2003), Chapter 6. 
 
 
 

 
Copyright © 2009 Global Association of Risk Professionals         82    
All rights reserved.
2009 FRM Practice Exams 

27. According to put‐call parity, buying a put option on a stock is equivalent to: 
 
a. Buying a call option and buying the stock with funds borrowed at the risk‐free rate. 
b. Selling a call option and buying the stock with funds borrowed at the risk‐free rate. 
c. Buying a call option, selling the stock and investing the proceeds  at the risk‐free rate. 
d. Selling a call option, selling the stock and investing the proceeds at the risk‐free rate. 
   
CORRECT: C 
 
Buying a call option, selling the stock and investing the proceeds  at the risk‐free rate. 
 
INCORRECT: A – Buying a call option is correct, but the rest of the statement is incorrect.  
INCORRECT: B – The entire statement is incorrect. 
INCORRECT: D – Selling a call option is incorrect, but the rest of the statement is correct.  
 
Reference: Options, Futures, and Other Derivatives, 6th edition, by John Hull, Chapter 10. 
 
 
 
 
28. A 3 month futures contract on an equity index is currently priced at USD 1000, the underlying index 
stocks are valued at USD 990 and pay dividends at a continuously‐compounded rate of 2 percent 
and the current continuously compounded risk‐free rate is 4 percent. The potential arbitrage profit 
per contract, given this set of data, is closest to 
 
a. USD 10.00 
b. USD 7.50 
c. USD 5.00 
d. USD 1.50 
 
CORRECT: C  
 
According to the fundamental pricing relationship between spot assets and the associated futures, 
the futures price, to prevent arbitrage, should equal 990 x e (0.04 – 0.02) x 0.25 or 995. Hence, the 
futures contract is overvalued, indicating it should be sold and the index should be purchased for an 
arbitrage profit of USD 1000 ‐ USD 995 = USD 5 
 
th 
Reference:  Hull, Options, Futures, and Other Derivatives, 6 ed. Chapter 5 
 
 
 
 

 
Copyright © 2009 Global Association of Risk Professionals         83    
All rights reserved.
2009 FRM Practice Exams 

29. Research and model projections indicate that a specific event is likely to move the CHF against the 
USD. While the direction of the move is highly uncertain, it is highly likely that magnitude of the 
move will be significant. Based on this information, which of the following strategies would provide 
the largest economic benefit? 
 
a. Long a call option on USD/CHF and short a put option on USD/CHF with the same strike price 
and expiration date 
b.  Long a call option on USD/CHF and long a put option on USD/CHF with the same strike price and 
expiration date 
c.  Short a call option on USD/CHF and long a put option on USD/CHF with the same strike price 
and expiration date 
d. Short a call option on USD/CHF and short a put option on USD/CHF with the same strike price 
and expiration date 
 
CORRECT: B 
 
The question tests on understanding of a “straddle” strategy and its application on currency trading. 
A long straddle strategy involves buying (long) a call and put option with the same strike price and 
expiration date, and will benefit most when the underlying moves away from the current equlibrium. 
Long call and long put create a straddle. 
 
INCORRECT: A – It sells a put option while it should buy one put. . 
INCORRECT: C – It sells a call option while it should buy one call.  
INCORRECT: D – It sells both the call and put option while it should buy both.  
 
Reference: Hull. 
 
 
 
30. Initially, the call option on Big Kahuna Inc. with 90‐days to maturity trades at USD 1.40. The option 
has a delta of 0.5739. A dealer sells 200 call option contracts and to delta‐hedge the position, the 
dealer purchases 11,478 shares of the stock at the current market price of USD 100 per share. The 
following day, the prices of both the stock and the call option increase. Consequently, delta 
increases to 0.7040. To maintain the delta hedge, the dealer should: 
 
a. Purchase 2,602 shares. 
b. Sell 2,602 shares. 
c. Purchase 1,493 shares. 
d. Sell 1,493 shares. 
 
CORRECT: A 
 
Number of calls = 200 contracts x 100 = 20,000 calls. 
 

 
Copyright © 2009 Global Association of Risk Professionals         84    
All rights reserved.
2009 FRM Practice Exams 

Number of shares   = (Number of calls) x (New delta – Old delta) 
      = 20,000 x (0.7040 – 0.5739) 
      = +2,602 shares 
Positive sign indicates that the manager should purchase new shares. 
 
INCORRECT: B – Because the formula is incorrect, i.e. old delta minus new delta. 
INCORRECT: C – Because the number of shares (instead of number of calls) is used in the calculation. 
INCORRECT: D – As per explanation in ‘C’ above and sign error. 
 
Reference: Hull. 
 
 
 
 
31. Which of the following strategies creates a calendar spread? 
 
a. Sell a call option with a certain strike price and buy a longer maturity call option with the same 
strike price. 
b. Buy a call option with a certain strike price and buy a longer maturity call option with the same 
strike price. 
c. Sell a call option with a certain strike price and buy a shorter maturity call option with the same 
strike price. 
d. Buy a call option with a certain strike price and sell a longer maturity call option with the same 
strike price. 
 
CORRECT: A 
 
INCORRECT: B – As buy a call option. 
INCORRECT: C – As buy a shorter‐maturity call option 
INCORRECT: D – As this is a reverse calendar spread. 
 
Reference: Hull. 
 

32. Which of the following underlying macro‐economic conditions would leave an emerging market 
most vulnerable to the contagion effects of a currency crisis?  
 
a. Large current account surplus, low foreign exchange reserves, non‐convertible currency 
b. Large current account deficit, low foreign exchange reserves, fully convertible currency 
c. Small current account deficit, high foreign exchange reserves, non‐convertible currency 
d. Large current account surplus, high foreign exchange reserves, fully convertible currency 
 
CORRECT: B 

 
Copyright © 2009 Global Association of Risk Professionals         85    
All rights reserved.
2009 FRM Practice Exams 

INCORRECT: A – Large current account surplus and non‐convertible currency would protect the local 
currency 
INCORRECT: C – High foreign exchange reserves and non‐convertible currency would protect the local 
currency 
INCORRECT: D – Large current account surplus and high foreign exchange would protect the local 
currency 
 
Reference: Saunders, Chapter 15, Foreign Exchange Risk 
 

33. Consider an FRA (forward rate agreement) with the same maturity and compounding frequency as a 
Eurodollar futures contract. The FRA has a LIBOR underlying. Which of the following statements are 
true about the relationship between the forward rate and the futures rate? 
 
a. They should be exactly the same. 
b. The forward rate is normally higher than the futures rate. 
c. The forward rate is normally lower than the futures rate. 
d. They have no fixed relationship. 
 
CORRECT: C 
 
As Eurodollar futures contract is marked to market and settled daily, normally forward rate is 
adjusted lower, so called convexity adjustment, by: 
1
Forward rate = Futures rate –  σ 2T1T2    
2
Reference: Hull. 
 
 
 
34. Your bank is an active player in the commodity market. The view of the economist of the bank is 
that inflation is expected to rise moderately in the near term and market volatility is expected to 
remain low. The traders are advised to undertake deals on the metals exchange to align your book 
to conform with the expectations of the economist of the bank. As risk manager, you are asked to 
monitor the positions of the traders to make sure that they have the exposures to inflation and 
market volatility sought by the bank. Which trader has taken an appropriate position among the 
traders you are monitoring?  
 
a. Trader A bought a call and a put, both with 90‐days to expiration and with strike price equal to 
the existing spot level. 
b. Trader B bought a put option with a down‐and‐in knock in feature. 
c. Trader C bought a call option at the existing spot levels and sold a call at a higher strike price, 
both with 90‐days to expiration. 
d. Trader D sold a call and bought a put at the existing levels, both with 90‐days to expiration. 
 

 
Copyright © 2009 Global Association of Risk Professionals         86    
All rights reserved.
2009 FRM Practice Exams 

CORRECT: C 
 
As the strategy popularly known as the bull spread will result in positive payoff when the spot rises. 
As inflation increases, spot levels in commodities are expected to rise. Selling a call at higher level 
will reduce the cost of the strategy. Although it may limit the upside, but that would be in line with 
the view as only a moderate rise is expected in spot. 
 
INCORRECT: A – As the strategy popularly known as a straddle is to be used when the view is that the 
volatility in the market will rise, and there is no directional view on the spot. 
INCORRECT: B – As the above option will be suitable when the spot is expected to fall from the 
existing levels. 
INCORRECT: D – As the payoff in this case is similar to short position in spot and would make sense 
when the underlying is expected to fall. 
 
Reference: Hull, Chapter 10. 
 
 
 
35. Considering options generally (i.e., not only plain vanilla calls and puts), which of the following 
statements about vega is correct?  
 
a. An option holder can never be vega negative. 
b. A deep in the money up and out call option has a negative vega. 
c. A deep out of money up and out call option has a negative vega. 
d. A deep out of money digital option has a negative vega.  
 
CORRECT: B 
 
 Deep in the money Up and Out call option because an increase in the volatility of such options leads 
to the increasing chances of option either being knocked out (if the price increases beyond the 
barrier) or loosing its moneyness (if the prices falls) and hence the increasing volatility tends to have 
negative impact on the price of the option.  
 
INCORRECT: A – As an option holder can be Vega negative as shown above. 
INCORRECT: C – have positive Vega as an increase in the volatility would increase the chances of 
getting towards moneyness and hence positive Vega from a holder’s perspective. 
INCORRECT: D – have positive Vega as an increase in the volatility would increase the chances of 
getting towards moneyness and hence positive Vega from a holder’s perspective. 
 
Reference: Hull, Chapters 17,18 24. 
 
 
 

 
Copyright © 2009 Global Association of Risk Professionals         87    
All rights reserved.
2009 FRM Practice Exams 

36. To hedge against future, unanticipated, and significant increases in borrowing rates, which of the 
following alternatives offers the greatest flexibility for the borrower? 
 
a. Fixed for floating swap 
b. Interest rate collar 
c. Interest rate floor 
d. Call swaption 
 
CORRECT: D 
 
The question focuses on flexible management of borrowing expenses.  While a fixed for floating 
swap could reduce borrowing expenses, it is a long‐term contractual commitment to exchange 
payments.  If interest rates decline, the borrower may gross up to the agreed fixed rate. An interest 
rate collar is a combination of an interest rate floor and cap, i.e., it locks in the interest expenses 
within a tight range.  Moreover, collars usually offer interest rate protection at one particular point 
of time unless several contracts with different maturities are exchanged. A call swaption gives the 
company the right to enter into a swap when the borrowing expenses exceed a certain reference 
rate.  If the reference rate is below the borrowing expenses, the option is not exercised. 
 
Reference: Hull. 
 
 
 
 
37. Assuming other things constant, bonds of equal maturity will still have different DV01 per USD 100 
face value.  Their DV01 per USD 100 face value will be in the following sequence of highest value to 
lowest value:   
 
a. Zero coupon bonds, par bonds, premium bonds 
b. premium bonds, par bonds,  zero coupon bonds  
c. Premium bonds, zero coupon bonds, par bonds    
d. Zero coupon bonds, premium bonds, par bonds 
 
CORRECT:  B   
 
DV01 is certain multiple of Dirty Price (which includes Coupons) and not Clean Price.  Thus, it is 
proportional to Base Price, which is Dirty Price.  Ordinarily, Premium Bond will have the highest 
(dirty) price followed by Par Bond and with the least price of Zero Coupon Bond.  Hence, DV01 of 
Premium Bond is the highest while that of Zero Coupon Bonds is the lowest.   
 
INCORRECT: A – Premium Bond will have a higher Base Price and hence higher DV01 than that of 
Zero Coupon Bond.  
INCORRECT: C – Base Price of Par Bond is higher than that of Zero Coupon Bond and hence, its DV01 
cannot be less than that of Zero Coupon Bond.   

 
Copyright © 2009 Global Association of Risk Professionals         88    
All rights reserved.
2009 FRM Practice Exams 

INCORRECT: D – DV01 per USD 100 Face Value is an Absolute Amount of USD based on actual Base 
Price Change.  Ordinarily, Base Price of a Zero Coupon Bond will be lower than that of Par & Premium 
Bond.   Hence, DV01 of Zero Coupon Bond is less than that of Premium Bond of same maturity.   
 
Reference: Tuckman, “Fixed Income Securities”, Chapter 5. 
 
 
 
38. The information ratio of the Sterole US Fund for 2006 against the S&P 500, its benchmark index, is 1.  
For the same time period, the fund’s Sharpe ratio is 2, the fund has a tracking error of 7% against 
the S&P 500, and the standard deviation of fund returns is 5%. The risk‐ free rate in the US is 4%. 
Calculate the return for the S&P 500 during the time period. 
 
a. 3.5% 
b. 7% 
c. 11% 
d. 14% 
 
CORRECT: B 
 
Sharpe Ratio = 2 
(Fund Return – Risk Free Rate)/SD = 2 
(Fund Return – 4%)/5% = 2 
Fund Return = 14% 
 
Information Ratio = 1 
(Fund Return – S&P 500 Return)/ Tracking Error = 1 
(14% ‐ S&P 500 Return) / 7% = 1 
S&P 500 Return = 7% 
 
INCORRECT: A – Incorrectly divides S&P 500 Return by 2 
INCORRECT: C – The candidate might use the Tracking Error as the Numerator in both the Ratios. 
Sharpe Ratio = 2 
(Fund Return – Risk Free Rate)/Tracking Error = 2 
(Fund Return – 4%)/7% = 2 
Fund Return = 18% 
Information Ratio = 1 
(Fund Return – S&P 500 Return)/ Tracking Error = 1 
(18% ‐ S&P 500 Return) / 7% = 1 
S&P 500 Return = 11% 
INCORRECT: D – The candidate can stop with the fund return calculation, and end up with 14%.   
Sharpe Ratio = 2 

 
Copyright © 2009 Global Association of Risk Professionals         89    
All rights reserved.
2009 FRM Practice Exams 

(Fund Return – Risk Free Rate)/SD = 2 
(Fund Return – 4%)/5% = 2 
Fund Return = 14% 
 
Reference: Amenc and Le Sourd, Portfolio Theory and Performance Analysis. Chapter 4 
 
 
 
 
39. A fund manager recently received a report on the performance of his portfolio over the last year. 
According to the report, the portfolio return is 9.3%, with a standard deviation of 13.5%, and a beta 
of 0.83. The risk‐free rate is 3.2%, the semi‐standard deviation σL(Rp) of the portfolio is 8.4%, and the 
tracking error of the portfolio to the benchmark index is 2.8%. What is the difference between the 
value of the fund’s Sortino ratio  (computed relative to the risk‐free rate) and its Sharpe ratio? 
 
a. 0.274 
b. 1.727 
c. 0.653 
d. ‐0.378 
 
CORRECT: A 
R p − RF 9 .3 % − 3 .2 %
Sharpe ratio equals to  = = 0.452  
σ (R p ) 13.5%
R p − RF 9.3% − 3.2%
While Sortino ratio equals to  = = 0.726  
σ L ( RP ) 8.4%
Tracking error is used to calculate the value of the information ratio, which is defined as 
R p − RB
, The calculation of information ratio is not required in this question. 
σ ( RP − RB )
 
INCORRECT: B –   2.178 – 0.452 = 1.727 
INCORRECT: C –   0.726 – 0.0.73 = 0.653  (0.073 = (.093 – 0.032)/0.83) 
INCORRECT: D –   0.073 – 0.452 = ‐0.378 
 
Reference: Amenc and Le Sourd, Portfolio Theory and Performance Analysis. Chapter 4 
 
 
 
 

 
Copyright © 2009 Global Association of Risk Professionals         90    
All rights reserved.
2009 FRM Practice Exams 

40. Your firm has no prior derivatives trades with its counterparty Super Bank. Your boss wants you to 
evaluate some trades she is considering. In particular, she wants to know which of the following 
trades will increase your firm’s credit risk exposure to Super Bank: 
I.             buying a put option 
II. selling a put option 
III. buying a forward contract 
IV. selling a forward contract 
 
a. I. and II only 
b. II and IV only 
c. III and IV only 
d. I, III and IV only 
 
CORRECT: D 
 
This tests understanding of the type of positions that create credit risk (and links to the following 
question).  The key is to evaluate each of the component trades.  Buying a put option creates credit 
risk.  Buying or selling forward contracts creates credit risk.  However, selling an option does not 
create credit risk you are not subject to the performance of the counterparty.  
 
Reference: Hull Chapter 22, Stulz Chapter 18. 
 
 
 
41. Consider the following one‐period transition matrix: 
 
Initial    Next Period State  
Period    A  B  Default
State  A  95% 5%  0% 
B  10% 80% 10% 
  Default 0%  0%  100% 
 
If a company is originally in State A, what is the probability that the company will have defaulted 
strictly before the fourth transition period from now? 
 
a. 0.875% 
b. 0.500% 
c. 1.375% 
d. 1.875% 
 
CORRECT:  C 

 
Copyright © 2009 Global Association of Risk Professionals         91    
All rights reserved.
2009 FRM Practice Exams 

The easiest way to determine the answer would be to make this a square matrix including default in 
initial state.  Then self‐multiplying the matrix three times yields three‐period transition matrix.  We 
can also manually do the calculation; 
After year 1 there is a 0% chance of default and 5% chance of being in state B.   
After year 2 there is 95%*5% + 80%*5% chance of being in state B and 5% * 10% chance of default.  
After year 3 there is a  (95%*5% + 80%*5%)*10% additional chance of default.  Answer A assumes 
just one year 
 
INCORRECT: A – Only considers the third year transition from B to default. 
INCORRECT: B – Only considers the second year transition from B to default. 
INCORRECT: D – Mistakenly doubles the second year transition from B to default. 
 
Reference: De Servigny and Renault, Measuring and Managing Credit Risk, Chapter 2, Appendix 2A, 
page 49 – 52. 

42. As an approximation, it is true that 
 
a. Default swap spread = Return of a risky bond + Return of a risk‐free bond  
b. Default swap spread = Return of a risky bond – Return of a risk‐free bond  
c. Default swap spread = Return of a risky bond x Return of a risk‐free bond  
d. Default swap spread = Return of a risky bond x (1 ‐ Return of a risk‐free bond)   
 
CORRECT: B 
 
The buyer of a risky bond can hedge the credit risk of the risky bond using a default swap. Entering 
into the swap trade reduces credit risk.  To preclude arbitrage, the buyer of the risk bond has to 
receive the same return as the risk‐free asset, or: 
Return of a risky bond = Return of a risk‐free bond + Default swap spread 
 
Reference: Hull, Chapter 23. 
 
 
 
43. In a CDO, the SPV is typically 
 
a. AAA‐rated   
b. A‐rated 
c. BBB‐rated  
d. Not rated 
 
CORRECT: A 
 

 
Copyright © 2009 Global Association of Risk Professionals         92    
All rights reserved.
2009 FRM Practice Exams 

In a CDO transaction, the Special Purpose Vehicle are special entities of financial institutions and are 
usually AAA rated. The SPV and the institution are legally distinct and credit quality deterioration of 
the financial institution does not affect the SPV. In this case SPV counterparty risk is low, which is 
desired by the investor. 
 
Reference: Hull chapter 23, Culp Chapters 16, 17, 18. 
 
 
 
 
44. A trader whose risk you are monitoring tells you that he wants to benefit from a credit spread 
widening due to a recession.  Which of the following would be good trades for his strategy? 
 
a. Go long risky bonds and short risk‐free bonds at the beginning of the recession. 
b. Short risky bonds and go long risk‐free bonds at the beginning of the recession. 
c.  Sell credit default swaps on bonds with a low credit quality and buy credit default swaps on 
bonds with a higher credit quality at the beginning of the recession. 
d. Sell credit default swaps on bonds with low credit quality and go long low credit quality bonds. 
 
CORRECT: B 
 
Shorting risky bonds and going long in risk‐free bonds at the beginning of the recession is the correct 
answer. During a recession, credit spreads typically start widening, and financing a long position in 
risk‐free bonds through declining credit quality risky bonds reduces the effective financing cost. 
 
INCORRECT: A – Going long in risky bonds and shorting risk‐free bonds at the beginning of the 
recession is incorrect.  This strategy is preferable at the beginning of an economic expansion, when 
the credit spread typically starts tightening.  
 
INCORRECT: C – Selling credit default swaps on bonds with a certain credit quality and buying credit 
default swaps on bonds with a higher credit quality at the beginning of the recession would be 
preferable at the beginning of an economic expansion.  
 
INCORRECT: D –  Selling credit default swaps on bonds with low credit quality and going long in low 
credit quality bonds effectively magnifies the credit risk, which under deteriorating credit conditions 
should be avoided. 
 
Reference: Culp, Chapter 12, 13, 16. 
 
 
 
 
 

 
Copyright © 2009 Global Association of Risk Professionals         93    
All rights reserved.
2009 FRM Practice Exams 

45. Bank B has a EUR 100 million loan portfolio and has set aside a reserve to cover the first EUR 20 
million in default‐related losses. If the bank wants to acquire protection for the remaining EUR  80 
million in risk exposure, which of the following solutions would work and would expose the bank to 
the least amount of counterparty risk? 
 
a. Buy credit protection in a senior subordinated CDS that covers EUR  80 million in losses above 
the first EUR 20 million.   
b. Buy credit insurance for losses up to EUR 80 million in excess of EUR 20 million on the loan 
portfolio. 
c. Issue a credit‐linked note in which interest and principal may be withheld from investors to 
cover up to EUR 80 million in losses above the first EUR 20 million on the loan portfolio. 
d. All three of the above choices work and expose the bank to the same amount of counterparty 
risk. 
 
CORRECT: C 
 
Both CDS and insurance are “unfunded,” and expose the bank to the risk of non‐performance by the 
CDS protection seller or the insurance company offering credit insurance. When issuing a credit‐
linked note, however, the cash has been paid in up‐front by investors to the bank, eliminating 
counterparty risk. 
 
Reference: Culp, Chapter 16 
 
 
 
 
46. Mr. Rosenqvist, Asset Manager at BCD Bank, holds a portfolio of SEK 200 million. The portfolio 
consists of BBB‐rated bonds. Assume that the one‐year probability of default is 4%, the recovery 
rate is 60%, and defaults are uncorrelated over years. What is the 2‐year cumulative expected credit 
loss on Mr. Rosenqvist’s portfolio? 

 
a. SEK 6.35 million 
b. SEK 6.40 million 
c. SEK 9.48 million 
d. SEK 9.60 million 
 
CORRECT: A 
 
The Credit Loss Year 1 is SEK million [200 * 4% * (1 – 60%)] = 3.2 and the Credit Loss Year 2 is SEK 
million [(200 – 3.2) * 4% * (1 – 60%)] = 3.15. The cumulative expected loss over 2 years than is 3.2 + 
3.15 = 6. 35. 
 
INCORRECT: B – Does not take into account the credit loss year 1 when calculating the loss for year 2.  

 
Copyright © 2009 Global Association of Risk Professionals         94    
All rights reserved.
2009 FRM Practice Exams 

INCORRECT: C – Wrongly interprets the recovery rate as a measure of credit loss, while credit loss 
equals (1 – recovery rate). In this case it equals (1 ‐ 0.60) = 40%.  
INCORRECT: D – Is a combination of mistake B and C. 
 
Reference: de Servigny and Renault, Measuring and Managing Credit Risk. 
 
 
 
 
47. Using the Merton model, the value of the debt increases if all other parameters are fixed and 

 
I.             The value of the firm decreases 
II. The riskless interest rate decreases 
III. Time to maturity increases 
IV. The volatility of the firm value decreases 
 
a. I and II only 
b. I and IV only 
c. II and III only 
d. II and IV only 
 
CORRECT: D 
 
According to the model, the value of the bond is B = V – S, where V is the value of the assets and S is 
the value of the equity, or Ke‐rtN(d2) + V x (1‐N(d1)). d1=ln(V/Ke‐rt)/σ√T+ σ√T/2 and d2 = 1 – d1. Value 
of the debt will increase if interest rates decreases and volatility of the firm decreases. 
 
INCORRECT: A – II is true but I is false. Value of the debt will increase if value of the firm increases. 
The value of the debt will increase if interest rate decreases. 
INCORRECT: B – V is true but I is false. 
INCORRECT: C – II is true but III is false. The value of the debt will increase of the time to maturity 
decreases 
 
Reference:Stulz, Risk Management & Derivatives, Chapter 18, p. 580  
de Servigny and Renault, Measuring and Managing Credit Risk. 
 
 
 
48. A firm is going to buy 10,000 barrels of West Texas Crude Oil.  It plans to hedge the purchase using 
the Brent Crude futures contract.  The correlation between the spot and futures prices is 0.72.  The 
volatility of the spot price is 0.35 per year.  The volatility of the Brent Crude futures price is 0.27 per 
year.  What is the hedge ratio for the firm? 
 

 
Copyright © 2009 Global Association of Risk Professionals         95    
All rights reserved.
2009 FRM Practice Exams 

a. 0.5554 
b. 0.9333 
c. 1.2099 
d. 0.8198 
 
CORRECT: B 
 
⎛ 0.35 ⎞
N = 0.72 × ⎜ ⎟
⎝ 0.27 ⎠  
N= 0.9333
 
INCORRECT: A – Inverts the spot volatility and the futures volatility. 
INCORRECT: C – Uses variances 
INCORRECT: D – Uses square roots of the volatilities. 
 
Reference:  Hull, Chapter 3 
 
 
 
 
49. It is June 2nd and a fund manager with USD 10 million invested in government bonds is concerned 
that interest rates will be highly volatile over the next three months.  The manager decides to use 
the September Treasury bond futures contract to hedge the value of the portfolio.  The current 
futures price is 95.0625.  Each contract is for the delivery of USD 100,000 face value of bonds.  The 
duration of the manager’s bond portfolio in three months will be 7.8 years.  The cheapest to deliver 
bond in the Treasury bond futures contract is expected to have a duration of 8.4 years at maturity of 
the contract.  At the maturity of the Treasury bond futures contract, the duration of the underlying 
benchmark Treasury bond is 9 years. What position should the fund manager undertake to mitigate 
his interest rate risk exposure? 
 
a. Short 94 contracts 
b. Short 98 contracts 
c. Short 105 contracts 
d. Short 113 contracts 
 
CORRECT: B 
 
⎛ 10,000,000 ⎞ ⎛ 7.8 ⎞
N=⎜ ⎟×⎜ ⎟
⎝ 95,062.50 ⎠ ⎝ 8.4 ⎠
 
N = 97.68 or 98 contracts
 
INCORRECT: A – Is made up.   

 
Copyright © 2009 Global Association of Risk Professionals         96    
All rights reserved.
2009 FRM Practice Exams 

INCORRECT: C – Leaves out the durations 
INCORRECT: D – Inverts the durations. 
 
Reference:  Hull, Chapter 6. 
 
 
50. A bond trader has bought a position in Treasury Bonds with a 4% annual coupon rate on February 
15, 2015. The DV01 of the position is USD 80,000. The trader decides to hedge his interest rate risk 
with the 4.5% coupon rate Treasury Bonds maturing on May 15, 2017 which has a DV01 of .076 per 
USD 100 face value. To implement this hedge, approximately what face amount of the 4.5% 
Treasury bonds maturing on May 15, 2017 should the trader sell? 
 
a. USD 80,000 
b. USD 10,500,000 
c. USD 80,000,000 
d. USD 105,000,000 
 
CORRECT: D 
 
USD 105,000,000x.076/100 = USD 79,800, which is pretty close to the desired DV01 of USD 80,000. 
To solve for the hedge, solve for F in the equation USD 80,000 = Fx.076/100, giving F = 105,263,158. 
 
INCORRECT: A – Selling this amount would offset a DV01 of only USD 80,000x.076/100 = USD 61. 
INCORRECT: B – USD 10,500,000x.076/100 = USD 7,980. 
INCORRECT: C – USD 80,000,000x.076/100 = USD 60,800. 
 
Reference: Tuckman  

 
 
END OF 2009 FRM FULL EXAM PRACTICE EXAM I 
Questions & Explanations  
 

 
Copyright © 2009 Global Association of Risk Professionals         97    
All rights reserved.

You might also like